Download Module 1. Clinical pharmacology of drugs affecting the

Document related concepts

Orphan drug wikipedia , lookup

Electronic prescribing wikipedia , lookup

Plateau principle wikipedia , lookup

Drug design wikipedia , lookup

Discovery and development of proton pump inhibitors wikipedia , lookup

Polysubstance dependence wikipedia , lookup

Stimulant wikipedia , lookup

Discovery and development of cephalosporins wikipedia , lookup

Psychedelic therapy wikipedia , lookup

Adherence (medicine) wikipedia , lookup

Drug discovery wikipedia , lookup

Bad Pharma wikipedia , lookup

Pharmacognosy wikipedia , lookup

Pharmacokinetics wikipedia , lookup

Bilastine wikipedia , lookup

Discovery and development of beta-blockers wikipedia , lookup

Pharmaceutical industry wikipedia , lookup

Prescription drug prices in the United States wikipedia , lookup

Medication wikipedia , lookup

Prescription costs wikipedia , lookup

Pharmacogenomics wikipedia , lookup

Psychopharmacology wikipedia , lookup

Drug interaction wikipedia , lookup

Neuropsychopharmacology wikipedia , lookup

Neuropharmacology wikipedia , lookup

Transcript
Module 1. Clinical pharmacology of drugs affecting the cardiovascular system and the respiratory
system. Clinical pharmacology of analgesics and antibacterials. Clinical pharmacology of
antiseptics, enzymes and drugs that affect the hard tissues of the tooth
Тext test tasks
1. Alternative antimicrobial drug(s) for treating penicillin-resistant S. aureus:
A. vancomycin (Vancocin)
B. * nafcillin (Nafcil, Unipen)
C. methicillin (Staphcillin)
D. All of the above
E. None of the above
2. Aminoglycosides antibacterial
A. hetacillin
B. aztreonam
C. * gentamicin (Garamycin)
D. vancomycin (Vancocin)
E. Clarithromicin
3. Antibacterials regarded as generally safe to prescribe in pregnancy.
A. * erythromycin
B. cephalosporins
C. tetracyclines
D. All of the above
E. None of the above
4. Antibiotic: Newborn toxicity due to impaired glucuronidation in infants
A. * chloramphenicol (Chloromycetin)
B. tetracycline (Achromycin)
C. ampicillin
D. all
E. neither
5. Antibiotic's clearance from the body most likely influenced by severe hepatic disease:
A. penicillins
B. clindamycin (Cleocin)
C. rifampin (Rimactane)
D. * clindamycin & rifampin
E. None of the above
6. Bacterial cell wall synthesis inhibitor--like penicillins:
A. bacitracin
B. lincomycin (Lincocin)
C. * cephalosporin
D. streptomycin
E. none of the above
7. Beta-lactamase inhibitor:
A. vancomycin (Vancocin)
B. sulbactam
C. clavulanate
D. * vancomycin & clavulanate
E. none of the above
8. Cephalosporin effective against pseudomonas infections:
A. cefoxitin (Mefoxin)
B. moxalactam
C. * ceftazidime (Fortax, Taxidime, Tazicef)
D. cefoxitin & ceftazidime
E. neither
9. Cephalosporin: orally effective
A. cephalothin (Keflin)
B. Cephalexin
C. * cefaclor (Ceclor)
D. cephalothin & cefaclor
E. none of the above
10. Combination of metronidazole (Flagyl) and penicillin in treating an abscess caused by betalactamase producing Bacteriodes and anaerobic streptococci is an example of:
A. synergistic drug treatment
B. antagonistic drug effects
C. additive drug effects
D. * none of the above
E. all of the above
11. Drug(s) effective in treating typhoid fever:
A. ampicillin (Principen, Omnipen)
B. ciprofloxacin (Cipro)
C. trimethoprim-sulfamethoxazole (Bactrim)
D. * all of the above
E. none of the above
12. Effective in treating infections due to B. fragilis
A. cefamandole (Mandol)
B. * cefoxitin (Mefoxin)
C. cefadroxil (Duricef, Ultracef)
D. all of the above
E. none of the above
13. FALSE statement about penicillin G
A. treatment of choice for viridans group streptococcal endocarditis.
B. * Pen G and Pen V are the two natural penicillins.
C. Pen G can be combined with procaine, extending drug half-life
D. Pen G most effective when given orally
E. None of the above
14. First generation cephalosporin:
A. cefamandole (Mandol)
B. cefaclor (Ceclor) and
C. cefoxitin (Mefoxin)
D. * cephalexin (Keflex)
E. none of the above
15. Fluoroquinone;effective in treating urinary tract infections (UTI) caused by multidrug
resistant strains.
A. isoniazid (INH)
B. sulfasalazine (Azulfidine)
C. * ciprofloxacin (Cipro)
D. streptomycin
E. none of the above
16. Inhibits bacterial cell wall synthesis:
A. streptomycin
B. vancomycin (Vancocin)
C. doxycycline (Vibramycin, Doryx)
D. * gentamicin (Garamycin)
E. all of the above
17. Mechanism of action: Chloramphenicol (Chloromycetin)
A. * protein synthesis inhibitor
B. bacterial cell wall synthesis inhibitor
C. DNA synthesis inhibition
D. none of the above
E. all of the above
18. Mechanism of action: vancomycin (Vancocin):
A. protein synthesis inhibitor
B. * cell wall synthesis inhibitor
C. inhibits folic acid synthesis
D. none of the above
E. all of the above
19. More likely to cause polyneuritis in patients who are"slow acelators"; this antibiotic is
inactivated by the liver.
A. clindamycin (Cleocin)
B. * isoniazid (INH)
C. nafcillin (Nafcil, Unipen)
D. vancomycin (Vancocin)
E. Flagyl
20. Properties of tetracycline (Achromycin):
A. inhibitor of bacterial cell wall synthesis
B. drug of choice in treating typhus
C. cleared primarily by the liver
D. * inhibitor of bacterial cell wall synthesis & drug of choice in treating typhus
E. none of the above
21. Safely administered at full doses to patients with renal insufficiency; has antimicrobial
activity against gram-positive bacteria.
A. gentamicin (Garamycin)
B. * erythromycin
C. neomycin
D. penicillin
E. all of the above
22. Suitable for treatment of bacterial meningitis caused by H. influenzae:
A. cefotaxime (Claforan)
B. cephalexin (Keflex)
C. cephalothin (Keflin)
D. * cefadroxil (Duricef, Ultracef)
E. none of the above
23. Synergistic antimicrobial combination:
A. amoxicillin-clavulanate
B. ampicillin -streptomycin
C. * sulbactam-ampicillin
D. All of the above
E. None of the above
24. Topical use only:
A. penicillin
B. * bacitracin
C. clindamycin (Cleocin)
D. ampicillin
E. none of the above
25. Unstable at low pH, this antibiotic is structurally similar to penicillin:
A. streptomycin
B. * erythromycin
C. trimethoprim (generic)
D. rifampin (Rimactane)
E. all of the above
26. Which of the following is a short acting leukotriene synthesis (5-lipoxygenase) inhibitor?
A. * Zileuton
B. Zafirlukast
C. Montelukast
D. Omalizumab
E. Theophyline
27. Generation of specific immunity requires antigen-presenting cells, APCs, including:
A. B lymphocytes
B. macrophages
C. Langerhans cells
D. B lymphocytes & macrophages
E. * B lymphocytes , macrophages & Langerhans cells
28. A client is taking famotidine (Pepcid) asks the doctor what would be the best medication to
take for a headache. The doctor tells the client that it would be best to take:
A. * aspirin (acetylsalicylic acid, ASA)
B. ibuprofen (Motrin)
C. acetaminophen (Tylenol)
D. naproxen (Naprosyn)
29. A doctor is caring for a client with a diagnosis of rheumatoid arthritis who is receiving
sulindac (Clinoril) 150 mg po twice daily. Which finding would indicate to the doctor that
the client is experiencing a side effect related to the medication?
A. * nausea
B. photophobia
C. fever
D. tingling in the extremities
E. All of the above
30. Anti-inflammatory drugs are:
A. prednisone (Deltasone)
B. aspirin
C. naproxen
D. prednisone & aspirin
E. * prednisone, aspirin & naproxen
31. Activated cytotoxic T cells induce target cell death (virus-infected cells) by:
A. Fas-Fas ligand apotosis pathway
B. lytic granule enzymes
C. perforin
D. * all of the above
E. None of the above
32. Adaptive Immune Response(s):
A. antibody production
B. lymphocyte activation
C. destroys proliferating lymphoid cells
D. * antibody production & lymphocyte activation
E. neither
33. Agent acts directly to diminish tissue injury:
A. tacrolimus
B. cyclosporine (Sandimmune, Neoral)
C. * prednisone (Deltasone)
D. dactinomycin (Cosmegen)
E. All of the above
34. Alkylating agent; destroys proliferating lymphoid cells; in low doses -- for effective
against autoimmune disorders including systemic lupus erythematosus
A. azathioprine (Imuran)
B. methotrexate
C. * cyclophosphamide (Cytoxan)
D. antilymphocyte globulin (ALG)
E. all the above
35. Analgesic effects of aspirin:
A. peripheral action (inflammation)
B. subcortical site of action
C. decreasing PGE2 synthesis
D. * all the above
E. neither
36. Antilymphocyte antibodies:
A. binds to the surface of T cells
B. acts mainly on small, long-lived peripheral lymphocytes
C. may be used in induction of immunosuppression
D. may be obtained by hybridoma technique for monoclonal antibody generation
E. * all of the above
37. Approximate prevalence of rheumatoid arthritis:
A. * 1%
B. 5%
C. 7%
D. 10%
E. 15%
38. Aspirin and antipyresis:
A. elevation of body temperature is typically a useful defense mechanism
B. * aspirin -- best available drug for reducing fever (in the absence of contraindications
to its use recurrent
C. aspirin -- more effective in lowering elevated temperature than normal body
temperature
D. aspirin-induced temperature reduction is caused by vasodilation
E. fever associated with infection: caused by actions of prostaglandins and interleukin 1
at the periphery
39. Aspirin may be fatal in taken in sufficient quantity. The syndrome of fatal salicyate
overdose in children is characterized by
A. marked hypotermia secondary to an antipyretic effect
B. * fever, tinnitus
C. peripheral oedema
D. disturbance in acid - base and electrolyte balance
E. * all of the above
40. Associated with elevated levels of antigen-antibody complexes, which cause tissue damage;
complement activation results in anaphylatoxic and chemotactic activities:
A. * Immediate Hypersensitivity Type III
B. Immediate Hypersensitivity Type II
C. Delayed Hypersensitivity
D. Immediate Hypersensitivity Type I
41. At low doses required to inhibition of thymidylate synthase, an enhanced adenosine release
A. chloroquine
B. gold
C. * methotrexate
D. cyclophosphamide
E. ketorolac
42. Autoimmune disorder: associated with antibodies made against self DNA, red blood
cells, histones, platelets
A. rheumatoid arthritis
B. * systemic lupus erythematosus
C. insulin dependent diabetes
D. all of the above
E. neither
43. Clinical uses of immunosuppressive drugs:
A. organ transplantation
B. hemolytic disease of the newborn
C. autoimmune disorders
D. *all of the above
E. neither
44. Common adverse reactions of corticosteroidal therapy are:
A. bradycardia, mental dullness
B. anorexia, polyuria
C. tachycardia, insomnia
D. * “moon face”, obese trunk
E. all of the above
45. Correct drug-drug interactions:
A. aspirin -- acetazolamide: enhanced salicylate intoxication
B. aspirin -- spironolactone: reduced spironolactone activity
C. *all of the above
D. aspirin -- phenytoin: increased free phenytoin serum levels
E. aspirin-alcohol: increased gastrointestinal bleeding
46. Cytotoxic agents with immunosuppressive properties:
A. cyclophosphamide (Cytoxan)
B. vincristine (Oncovin)
C. methotrexate
D. cytarabine (ARA-C)
E. * all the above
47. Drug associated with the hepatic/renal toxic metabolite: N-acetyl-p-benzoquinone
A. diclofenac
B. meclofenamate
C. indomethacin
D. * acetaminophen
E. aspirin
48. Drug of choice in treating autoimmune hemolytic anemia:
A. cyclophosphamide plus factor XIII
B. Rho(D) immune globulin
C. * prednisone
D. OKT3 monoclonal antibody
E. cyclosporine
49. Effective in lowering incidents of rejection in infectious complications in transplant patients:
A. azathioprine (Imuran)
B. prednisone (Deltasone)
C. antilymphocyte antibodies
D. cyclosporine (Sandimmune, Neoral)
E. * all the above
50. Effective in managing acute gouty arthritisand ankylosing spondylitis; also accelerates
closure of patent ductus arteriosus in premature infants:
A. gold
B. ketorolac
C. phenylbutazone
D. methotrexate
E. * indomethacin
51. Example of first-class of hormonal agents noted to have lipolytic properties:
A. cyclosporine
B. * prednisone (Deltasone)
C. tacrolimus
D. IFN-alpha
E. thalidomide
52. Example of immunostimulatory cytokine that may be useful in cancer immunotherapy
A. levamisole
B. BCG
C. IFN-alpha
D. * IL-2
E. none of the above
53. Examples of autoimmune diseases
A. rheumatoid arthritis
B. insulin-dependent diabetes mellitus
C. systemic lupus erythematosis
D. none of the above
E. * all of the above
54. For a 6-year-old child with fever, what NSAIDs would be preferred?
A. aspirin
B. * acetaminophen
C. indomethacin
D. ibuprofen
55. Glucocorticoid effects:
A. inhibition of leukotriene production
B. inhibition of prostaglandins
C. increased chemotaxis
D. * inhibition of leukotriene production & inhibition of prostaglandins
E. all of the above
56. In rheumatoid arthritis: primary effect of this mediator is on prostaglandin production:
A. PDGF (platelet-derived growth factor)
B. GM-CSF
C. * TNF alpha
D. all of the above
E. none of the above
57. Inhibits antigen recognition of B-cell
A. prednisone
B. azathioprine
C. methotrexate
D. * Rho(D) immune globulin
E. tacrolimus
58. Interferes with cell cycle of activated lymphoid cells;example of the first class of hormonal
agents recognized have lympholytic properties:
A. cyclosporine
B. tacrolimus
C. IFN-alpha
D. * dexamethasone
E. 15-desoxyspergualin
59. Isozyme primarily responsible for prostaglandin production by cells involve an
inflammation:
A. COX-I
B. * COX-II
C. COX-III
D. all of the above
E. none of the above
60. Mechanism of action: aspirin-platelet effects:
A. promotes platelet aggregation
B. activates thromboxane synthesis
C. decreasing PGE2 synthesis
D. all of the above
E. * neither
61. Mediator most likely to promote pain:
A. histamine
B. serotonin
C. * bradykinin
D. prostaglandins
E. leukotrienes
62. Mediator promoting greatest increase in vascular permeability, associated with acute
inflammation:
A. serotonin
B. prostaglandins
C. bradykinin
D. * leukotrienes
63. NSAID primarily promoted as an analgesic, not as an anti-inflammatory agent:
A. piroxicam
B. ibuprofen
C. naproxen
D. * ketorolac
E. sulindac
64. Reduces uric acid synthesis: for management of gout
A. colchicine
B. probenecid (Benemid)
C. sulfinpyrazone (Anturane)
D. * allopurinol (Zyloprim, Purinol)
E. oxaprozin (Daypro)
65. Structural analog/antimetabolite: cytotoxic immunosuppressive drug:
A. vincristine
B. cyclophosphamide
C. * azathioprine
D. All of the above
E. none of the above
66. T helper lymphocytes -- regulate each other; this substance produced by TH1cells inhibits
TH2 cellular proliferation:
A. interleukin-2
B. tumor necrosis factor beta
C. * interferon-gamma
D. IL-4
E. IL-6
67. The chemotherapeutic agent also used orally for severe forms of arthritis is
A. 5-fluorouracil (5-FU)
B. * methotrexate (Mexate)
C. cyclophosphamide (Cytoxan)
D. cisplatin (Platinol)
E. All of the above
68. The following is not true about corticosteroids:
A. have anti-inflammatory activity
B. highly lipophillic
C. decrease the quantity and viscosity of mucus secretions
D. * decrease the synthesis of adrenergic mediators
E. decrease the transcription of genes coding for pro-inflammatory cytokines
69. The following is not true about corticosteroids:
A. have anti-inflammatory activity
B. highly lipophillic
C. decrease the quantity and viscosity of mucus secretions
D. * decrease the synthesis of adrenergic mediators
E. decrease the transcription of genes coding for pro-inflammatory cytokines
70. The major role in chronic management of arthritis:
A. * glucocorticoids
B. nonsteroidal antiinflammatory drugs
C. interferon-gamma
D. All of the above
E. None of the above
71. To the patient with gout the doctor prescribed allopurinol. What pharmacological
property of allopurinol is important in this situation?
A. * Competitive inhibition of ksantinoxydase
B. Increasing of evacuation of nitrogen substances
C. Acceleration of catabolism of pirimidinic nucleotides
D. Decreasing of reutilization of pirimidinic nucleotides
E. Increasing of synthesis of nucleic acids
72. Type 1 immediate hypersensitivity:
A. due to elevated levels of antigen-antibody complexes
B. * stems from cross-linking of membrane-bound IgE on blood basophils or tissue
mast cells by antigen
C. follows from formation of antigen- antibody complexes between foreign antigen and
IgM or IgG immunoglobulins
D. tissue damage due to influx of antigen-nonspecific inflammatory cells (macrophages
and neutrophils)
E. All above
73. Typically associated with B cell proliferation and differentiation into antibody-secreting
plasma cells
A. tumor necrosis factor beta
B. interferon-gamma
C. * IL-4 (interleukin 4)
D. IL-2 (interleukin 2)
E. all of the above
74. Useful in management of idiopathic thrombocytopenic purpura refractory to prednisone
A. dactinomycin
B. * vincristine
C. cyclophosphamide
D. azathioprine
E. all of the above
75. What adverse effects are associated with chronic use of systemic corticosteroids?
A. Candidosis
B. * Gastrointestinal complications
C. Hypotension
D. Hypoglycemia
E. all of the above
76. Which of the following combinations is false?
A. Rifampin & Light-chain proteinurea
B. Ethambutol & M.avium complex
C. Linezolide & MDR mycobacteria
D. Streptomycine & Brucellosis
E. * Para-amino Salicylic acid (PAS) & Optic neuropathy
77. A child has ingested an unknown substance and has evidence of respiratory depression. This
symptom is usually found with poisoning due to:
A. amphetamines
B. atropine
C. mushrooms
D. kerosene
E. * opioids
78. A drug which could help you confirm the patient's ingestion is:
A. phenylephrine
B. naloxone
C. * physostigmine
D. edrophonium
E. neostigmine
79. A good analgesic at subanesthetic doses Anesthetic Partition Minimum Alveolar Coefficient
Concentration (MAC)
A. halothane 2.30 0.75
B. enflurane 1.80 1.68
C. * nitrous oxide 0.47 105.00
D. isoflurane 1.40 1.15
E. methoxyflurane 12.00 0.16
80. A man is receiving oral hydromorphone (Dilaudid) 10 mg every 4 hours. He needs to be
NPO. What would be the best equivalent Intravenous dose of Dilaudid, administered as a
continuous infusion?
A. 0.1 mg per hour
B. * 0.5 mg per hour
C. 1.0 mg per hour
D. 1.5 mg per hour
E. 2.5 mg per hour
81. A patient you follow in clinic has a well-known heroin abuse problem. Drugs which could
potentially prevent an abstinence withdrawal syndrome during hospitalization include:
methadone, nalbuphine, butorphanol
A. morphine, nalbuphine, methadone
B. methadone, nalbuphine
C. nalbuphine, butorphanol
D. butorphanol only
E. * All of the above
82. A pharmacologic agent which has the potential to cause increased biliary tree pressure is:
A. * morphine
B. warfarin
C. phenytoin
D. acetazolamide
E. carbon tetrachloride
83. Acetaminophen has been used as a safe and effective analgesic/ antipyretic agent for over 80
years. Since it may be purchase without a prescription, it is readily available and as such the
recommended dose may be exceeded. The primary toxicity leading to death from an
acetaminophen overdose is:
A. papillary necrosis and chronic interstitial nephritis
B. pancytopenia
C. * hepatocellular necrosis
D. myocarditis
E. hemolytic anemia
84. Agents used to counter motion sickness which are more effective than cyclizine and
meclizine include: dimenhydramine, promethazine, scopolamine, bethanechol
A. * dimenhydramine, promethazine, scopolamine
B. dimenhydramine, scopolamine
C. promethazine, bethanechol
D. bethanechol only
E. All of the above
85. All of the following are common signs of approaching death except:
A. Decreasing interest in eating
B. Decreasing recognition of family members
C. Decreasing urine output
D. Increasing confusion
E. * Increasing number of bowel movements
86. All of the following classes of drugs can demonstrate anticholinergic side effects EXCEPT:
A. * tricyclic antidepressants
B. antihistamines
C. phenothiazines
D. anticonvulsants
E. All of the above
87. All of the following drugs are classified as indirect or mixed acting agents EXCEPT:
ephedrine, dopamine, amphetamine, dobutamine
A. ephedrine, dopamine, amphetamine
B. ephedrine, amphetamine
C. dopamine, dobutamine
D. * amphetamine only
E. all are correct
88. All of the following drugs are examples of direct acting beta agonists EXCEPT:
A. * phenylephrine
B. metaproterenol
C. isoetharine
D. albuterol
E. terbutaline
89. All of the following factors influence the rate of induction of anesthesia with an inhaled
anesthetic EXCEPT:
A. aqueous solubility of the anesthetic
B. * patient history of malignant hyperthermia
C. ventilation rate
D. tension of gas administration
E. pulmonary blood flow rate
90. All of the following statements are true concerning effects of intravenous infusion of
norepinephrine at 10 UG/Min EXCEPT:
A. * an increase in pulse rate
B. an increase in pulse pressure
C. a marked increase in peripheral resistance
D. an increase in systolic blood pressure
E. an increase in diastolic blood pressure
91. All of the following statements concerning catecholamine biosynthesisare true EXCEPT:
A. * all of the enzymes involved with catecholamine biosynthesis are highly specific for
each substance
B. tyrosine hydroxylase is the rate-limiting enzyme in catecholamine biosynthesis
C. the level of glucocorticoid influences the concentration of phenyl-nmethyltransferase and hence the conversion of norepinephrine to epinephrine
D. the conversion of tyrosine to dopa and dopa to dopamine occurs in the cytoplasm of
the adrenergic nerve terminal
E. the conversion of dopamine to norepinephrine via dopamine beta-hydroxylase tales
place within the granule of the adrenergic nerve terminal
92. All of the following statements concerning physostigmine and neostigmine are true
EXCEPT:
A. physostigmine and neostigmine are classified as reversible cholinesterase inhibitors
B. neostigmine stimulates nicotinic receptors at ganglia and neuromuscular junctions
unlike physostigmine
C. physostigmine is well absorbed orally
D. * a major problem encountered with neostigmine is the unpleasant central nervous
system effects which result from its administration
E. physostigmine is a tertiary amine and is able to penetrate the central nervous system
93. All of the following statements concerning the effects of anticholinesterase agents are true
EXCEPT:
A. when applied topically to the anti-cholinesterase agents cause conjunctival
hyperemia, miosis, and constriction of the ciliary muscle
B. enhancement of gastric contractions and increased secretion of gastric acid from
parietal cells
C. augmentation of secretory function in bronchial, lacrimal, sweat, salivary, gastric,
and intestinal glands
D. * increase the effective refractory period of cardiac muscle and decrease the
conduction time in conducting tissue
E. stimulation followed by depression or paralysis, of all autonomic ganglia and
skeletal muscle (nicotinic action)
94. All of the following statements concerning the structure-activity relationships of
sympathomimetic amines are true EXCEPT:
A. the presence of a hydroxyl group in the 3 and 4 positions on the benzene ring is
characteristic of the "catechol" nucleus
B. resistance to monamine oxidase is conferred by alkyl substitution on the alpha
carbon of the phenylethylamine structure
C. * the presence of hydroxyl groups in the 3 and 5 positions of a benzene ring
increases the specificity for beta-1 receptors in the lung
D. a large alkyl substitution on the amino group of a direct-acting sympathomimetic
increases the specificity for beta receptors
E. the absence of hydroxyl groups of the benzene ring of indirect-acting
sympathomimetic amines increases oral effectiveness
95. All of the following statements regarding acetaminophen toxicity are true EXCEPT:
A. acetaminophen is the principal metabolic product of phenacetin
B. an overdose of 10 grams or more may produce hepatic necrosis in adults
C. acetaminophen overdose may produce transient azotemia or renal failure
D. * forced diuresis may be a useful form of therapy for acetaminophen overdose
E. liver pathology is centrilobular and midzonal necrosis
96. All of the following statements regarding cholinergic blockage are true EXCEPT:
A. muscarinic actions of all parasympathomimetic drugs are blocked selectively by
atropine, through competitive occupation of cholinergic receptors
B. epinephrine and other sympathomimetic amines antagonize most muscarinic effects
at sites where adrenergic and cholinergic impulses produce opposing effects
C. nicotinic effects of acetylcholine and its derivatives at autonomic ganglia are blocked
by hexamethonium
D. * nicotinic effects of acetylcholine and its derivatives at the neuromuscular junction
of skeletal muscle are antagonized by D-tubocurarine
E. muscarinic effects of acetylcholine and its derivatives at autonomi ganglia are
blocked by hexamethonium
97. Anesthetic associated with the highest incidence of hepatitis. Anesthetic Partition Minimum
Alveolar Coefficient Concentration (MAC)
A. * halothane
B. enflurane
C. nitrous oxide
D. isoflurane
E. methoxyflurane
98. Anesthetic frequently used for analgesia during surgery. Large doses induce or maintain
anesthesia but mechanical ventilation required due to profound central respiratory
depression.
A. thiopental
B. propofol
C. ketamine
D. * etomidate
E. fentanyl
99. Anesthetic that must be administered in a lipid diluent due to poor aqueous solubility.
A. thiopental
B. * propofol
C. ketamine
D. etomidate
E. fentanyl
100.
Aspirin is a nonopioid analgesic which is thought to work by inhibiting:
prostaglandin reductase, prostaglandin synthetase, thromboxane synthetase, cyclooxygenase
A. prostaglandin reductase, prostaglandin synthetase, thromboxane synthetase
B. prostaglandin reductase, thromboxane synthetase
C. * prostaglandin synthetase, cyclooxygenase
D. cyclooxygenase only
E. All of the above
101.
Aspirin may be fatal if taken in sufficient quantity. The syndrome of acute salicylate
overdose in children is characterized by: marked hypothermia secondary to an antipyretic
effect, fever, peripheral edema, disturbance in acid-base and electrolyte balance
A. fever, marked hypothermia secondary to an antipyretic effect, peripheral edema
B. marked hypothermia secondary to an antipyretic effect, peripheral edema
C. * fever, disturbance in acid-base and electrolyte balance
D. disturbance in acid-base and electrolyte balance only
E. All of the above
102.
At a daily dose of 240 mg of long-acting morphine, the most appropriate order for
rescue (breakthrough) short-acting morphine (MSIR) is:
A. 15 mg q 2h
B. * 30 mg q 2h
C. 15 mg q 4h
D. 30 mg q 4h
E. 45 mg q 4h
103.
Because of delayed onset of activity after the first application of a Duragesic Patch
(transdermal fentanyl), it is especially important to have breakthrough pain medicine
available for the first:
A. 2-6 hours
B. 7-12 hours
C. * 13-24 hours
D. 24-36 hours
E. 48-72 hours
104.
Characteristically observed in individuals following acute overdose of opioids.
A. * pinpoint pupils
B. depressed respiration
C. coma
D. convulsions
E. All of the above
105.
Chronic renal damage resulting from the ingestion of analgesics has been suggested.
The FALSE statement is:
A. phenacetin has been implicated
B. salicylates have been implicated
C. * meperidine has been implicated
D. acetaminophen is a metabolite of phenacetin
E. combinations may be more harmful
106.
Comfort measures during the Syndrome of Imminent Death include all the following,
except:
A. Anti-pyretic PRN fever
B. Changing body position frequently
C. * Intravenous hydration
D. Lip balm
E. Morphine PRN respiratory distress
107.
Compared to morphine, which one of the following opioids is more likely to result in
respiratory depression during dose escalation:
A. hydrocodone (e.g. Vicodin or Lortab)
B. hydromorphone (Dilaudid)
C. meperidine (Demerol)
D. * methadone (Dolophine)
E. oxycodone (e.g. Percocet)
108.
Correct statements regarding etomidate include all EXCEPT:
A. inhibits adrenocortical function
B. no adverse effects from adrenocortical inhibition during short-term use
C. produces minimal cardiovascular effects
D. * potent hypnotic with excellent analgesic properties
E. safe alternative for a patient with unstable cardiovascular status
109.
Correct statements regarding the D-2 receptor include: the ergot alkaloids act as
agonists at the receptor; the D-2 Receptor is generally less sensitive to dopamine agonists
than is the D-1 receptor; metoclopramide is felt to be a selective antagonist at the D-2
receptor; the prototype location for the D-2 receptor is the parathyroid gland
A. the ergot alkaloids act as agonists at the receptor, the D-2 Receptor is generally less
sensitive to dopamine agonists than is the D-1 receptor, metoclopramide is felt to be
a selective antagonist at the D-2 receptor
B. * the ergot alkaloids act as agonists at the receptor, metoclopramide is felt to be a
selective antagonist at the D-2 receptor
C. the D-2 Receptor is generally less sensitive to dopamine agonists than is the D-1
receptor only
D. metoclopramide is felt to be a selective antagonist at the D-2 receptor only
E. All of the above
110.
Correct statements regarding the D-2 receptor include: A. the ergot alkaloids act as
agonists at the receptor; B. the D-2 receptor is generally less sensitive to dopamine agonists
than is the D-1 receptor; C. metoclopramide is felt to be a selective antagonist at the D-2
receptor; D. the prototype location for the D-2 receptor is the parathyroid gland
A. the ergot alkaloids act as agonists at the receptor, the D-2 Receptor is generally less
sensitive to dopamine agonists than is the D-1 receptor, metoclopramide is felt to be
a selective antagonist at the D-2 receptor
B. * the ergot alkaloids act as agonists at the receptor, metoclopramide is felt to be a
selective antagonist at the D-2 receptor
C. the D-2 receptor is generally less sensitive to dopamine agonists than is the D-1
receptor only
D. metoclopramide is felt to be a selective antagonist at the D-2 receptor only
E. All of the above
111.
Death resulting from side effects of opioid analgesics, used with the intent to treat
severe dyspnea in a dying patient, is an example of:
A. * Acceptable medical practice
B. Assisted suicide
C. Euthanasia
D. Murder
E. Unprofessional practice
112.
Drug Y produces a slowing heart rate when injected intravenously into an animal.
This slowing in heart rate does not occur if the animal is bilaterally vagotomized. Drug Y
may be: phenylephrine; levarterenol; methoxamine; isoproterenol
A. * phenylephrine, levarterenol, methoxamine
B. phenylephrine, methoxamine
C. levarterenol, isoproterenol
D. isoproterenol only
E. All of the above
113.
Drugs which have no nicotinic activity, and are resistant to the activity of
acetylcholinesterase include: acetylcholine; methacholine; carbachol; bethanechol
A. acetylcholine, methacholine, carbachol
B. acetylcholine, carbachol
C. methacholine, bethanechol
D. bethanechol only
E. * All of the above
114.
Examples of pharmacologic effects of ganglionic blocking drugs include:
cycloplegia; vasodilatation, increased peripheral blood flow; hypotension; anhidrosis;
bradycardia
A. * cycloplegia, increased peripheral blood flow; hypotension, anhidrosis
B. cycloplegia, anhidrosis
C. bradycardia only
D. All of the above
115.
FALSE statement regarding ketamine as an anesthetic:
A. contraindicated in a patient with asthma because it may cause bronchoconstriction
B. * recovery from anesthesia associated with emergence phenomenon of hallucinations
and vivid dreams
C. produces dissociative anesthetic state, whereby patient appears awake but is
unconscious
D. produces analgesia
116.
False statement regarding propofol as an anesthetic:
A. pharmacokinetic half-life does NOT correlate with duration of central nervous
system depression
B. * provides adequate skeletal muscle relaxation alone
C. vasodilation may produce a decline in blood pressure. Can be minimized by
decreasing dose.
D. safe alternative for patients predisposed to malignant hyperthermia
E. high lipophilicity provides rapid and smooth onset, dose titratability, and rapid
recovery from anesthesia
117.
For patients on chronic opioids, the most appropriate equianalgesic conversion ratio
between oral and intravenous morphine is:
A. 3 mg oral = 18 mg intravenous
B. 3 mg oral = 9 mg intravenous
C. 3 mg oral = 3 mg intravenous
D. * 3 mg oral = 1 mg intravenous
E. 3 mg oral = 0.3 mg intravenous
118.
Ganglionic blockade would be expected to give rise to all of the following EXCEPT:
A. arteriolar vasodilation, increased peripheral flow and hypotension
B. venodilation, pooling of blood, decreased venous return, and decreased cardiac
output
C. reduced gastrointestinal tone and motility; constipation
D. * bradycardia
E. All of the above
119.
Histamine-1 receptor antagonists include:
A. ranitidine
B.
C.
D.
E.
nizatidine
famotidine
* diphenhydramine
All of the above
120.
In contrast to most anesthetics, this agent produces cardiac stimulation, resulting in
increased blood pressure, heart rate and cardiac output.
A. thiopental
B. propofol
C. * ketamine
D. etomidate
E. fentanyl
121.
Manifestations of overdose with diphenhydramine may include:
A. blurred vision
B. dry mouth
C. hallucinations
D. sinus tachycardia
E. * All of the above
122.
Many commonly utilized medications are subject to abuse. Normally nontoxic
medications, if ingested in excessive amounts, may cause end-organ damage. Analgesic
nephropathy has been associated with:
A. prolonged abuse of phenacetin
B. prolonged abuse of phenacetin-aspirin combinations
C. acetaminophen derived from phenacetin
D. * All of the above
E. None of the above
123.
Many stimuli may cause vomiting. The chemoreceptor trigger zone of the central
nervous system is:
A. located in the cerebral cortex
B. stimulated in all forms of vomiting
C. * stimulated by morphine and its congeners
D. All of the above
E. None of the above
124.
Morphine's affects the eye by:
A. * producing miosis through an action on the oculomotor nerve
B. producing mydriasis through an action on the sympathetic system
C. decreasing pupillary responses to light
D. directly acting on the smooth muscles of the iris
E. directly acting on extrinsic muscles of the eye
125.
Neuropathic pain is often characterized by episodes of:
A. Achy pain
B. Colicky pain
C. Gnawing pain
D. * Shock-like pain
E. Squeezing pain
126.
Neurotransmitters found within the central nervous system include: dopamine;
gamma-aminobutyric acid (GABA); 5-hydroxytryptamine (Serotonin); norepinephrine
A. dopamine, gamma-aminobutyric acid (GABA), 5-hydroxytryptamine (Serotonin)
B. dopamine, 5-hydroxytryptamine (Serotonin)
C. gamma-aminobutyric acid (GABA), norepinephrine
D. norepinephrine only
E. * All of the above
127.
Opioid analgesics are sometimes associated with the production of pulmonary
disease. The most common pulmonary complication after oral ingestion is:
A. * pulmonary edema
B.
C.
D.
E.
interstitial fibrosis
pulmonary calcification
bronchoconstriction
pleural effusion
128.
Pain that is described as “dull, achy” and is well localized, is best described as:
A. Autonomic pain
B. Neuropathic pain
C. * Somatic pain
D. Vascular pain
E. Visceral pain
129.
Patients with normal platelet counts and normal bleeding time may still bleed
severely as a result of aspirin ingestion prior to a dental or surgical procedure. The aspirin
interference with normal platelet function may last as long as:
A. 4 hours
B. 12 hours
C. 2 days
D. * 5 days
E. 7 days
130.
Patients with substance abuse and HIV:
A. Have pain that responds poorly to tricyclics or anticonvulsants
B. Should never receive opioids because of the risk of addictive behavior
C. Should not be treated with a fentanyl patch
D. Should only receive opioids on a PRN basis, not continuously
E. * Will likely need higher doses of opioids to achieve similar pain relief
131.
Recent investigation has demonstrated the existence of both presynaptic alpha
receptors (a2) and postsynaptic alpha receptors (a1). All of the following statements
concerning these alpha receptors are true EXCEPT:
A. clonidine is an a2 selective agonist
B. * methoxamine and phenylephrine are a2 selective agonists
C. prazosin and phenoxybenzamine are a1 selective antagonists
D. phentolamine is a nonselective alpha antagonist
E. epinephrine and norepinephrine are nonselective alpha agonists
132.
Respiratory effects of epinephrine include all of the following EXCEPT:
A. relaxation of bronchial smooth muscle
B. inhibition of antigen-induced release of histamine
C. potent central nervous system stimulation
D. * increase in respiratory rate and tidal volume
E. constriction of pulmonary vessels when administered as an aerosol
133.
Symptoms that might be helpful in distinguishing between 'cholinergic crisis'(e.g.,
overdose with cholinesterase inhibitors) and an acute exacerbation of myasthenia gravis
include: increased salivation; muscle fasciculations (twitching); profuse sweating; muscle
weakness
A. increased salivation, muscle fasciculations (twitching), profuse sweating
B. increased salivation, profuse sweating
C. muscle fasciculations (twitching), muscle weakness
D. muscle weakness only
E. * All of the above
134.
The analgesic most apt to produce dysphoria:
A. morphine
B. meperidine
C. methadone
D. codeine
E. * pentazocine
135.
The anesthetic that may be most safely used in a patient with a history of malignant
hyperthermia? Anesthetic Partition Minimum Alveolar Coefficient Concentration (MAC)
A. halothane
B. enflurane
C. * nitrous oxide
D. isoflurane
E. methoxyflurane
136.
The anesthetic which should be avoided in patients with a seizure disorder because
tonic-clonic seizures are associated with its use. Anesthetic Partition Minimum Alveolar
Coefficient Concentration (MAC)
A. halothane
B. * enflurane
C. nitrous oxide
D. isoflurane
E. methoxyflurane
137.
The blood:gas partition coefficient is the ratio of anesthetic concentration in blood
compared to gas phase. Greater solubility in blood results in slower onset of anesthesia and
higher partition coefficient value. Which anesthetic has the fastest onset of action?
Anesthetic Partition Minimum Alveolar Coefficient Concentration (MAC)
A. halothane
B. enflurane
C. * nitrous oxide
D. isoflurane
E. methoxyflurane
138.
The gastric mucosa has the important ability to prevent movement of gastric acid
from the stomach lumen into the gastric wall. Some diseases and drug regiments have been
implicated as causes of increased gastric mucosal permeability to hydrogen ion. Drugs
which may increase gastric wall permeability include:
A. erythromycin
B. indomethacin
C. nitrofurantoin
D. aspirin
E. * indomethacin & aspirin
139.
The halogenated anesthetic which is advantageous in patients with cardiovascular
disease because it maintains cardiac output, produces systemic and coronary vasodilation,
and catecholamine dependent arrhythmias are uncommon. Anesthetic Partition Minimum
Alveolar Coefficient Concentration (MAC)
A. halothane
B. enflurane
C. nitrous oxide
D. * isoflurane
E. methoxyflurane
140.
The intravenous anesthetic with excitatory effects on the central nervous system but
produces the least cardiovascular disturbance among intravenous anesthetics.
A. thiopental
B. propofol
C. ketamine
D. * etomidate
E. nitrous oxide
141.
The maximum recommended daily dose of acetaminophen is:
A. 2 grams
B. * 4 grams
C. 6 grams
D. 8 grams
E. 10 grams
142.
The most appropriate equianalgesic conversion ratio between IV morphine and IV
hydromorphone (Dilaudid) is:
A. 1 mg morphine = 8 mg hydromorphone
B. 1 mg morphine = 4 mg hydromorphone
C. 1 mg morphine = 1 mg hydromorphone
D. 1 mg morphine = 0.50 mg hydromorphone
E. * 1 mg morphine = 0.20 mg hydromorphone
143.
The most appropriate equianalgesic conversion ration between IV morphine and
Subcutaneous morphine is:
A. 1.0 mg IV = 0.25 mg SQ
B. 1.0 mg IV = 0.5 mg SQ
C. * 1.0 mg IV = 1.0 mg SQ
D. 1.0 mg IV = 2.0 mg SQ
E. 1.0 mg IV = 4.0 mg SQ
144.
The most important supplemental therapy to consider when starting patients on
opioids for pain is:
A. Amphetamines to increase alertness
B. Antidepressants to supplement pain relief
C. Antiepileptic medications to treat neuropathic pain
D. * Laxatives to prevent constipation
E. Non-steroidals (NSAID’s) to treat inflammation
145.
The opioid expected to have the shortest duration of action following subcutaneous
administration of equianalgesic doses:
A. hydromorphone
B. * meperidine
C. methadone
D. morphine
E. codeine
146.
The pharmacologic effects of morphine include all EXCEPT:
A. behavioral changes
B. miosis
C. respiratory depression
D. * diarrhea
E. postural hypotension
147.
The rate-limiting enzyme in catecholamine biosynthesis is:
A. aromatic l-amino acid decarboxylase
B. dopamine-beta-hydroxylase
C. tryptophan hydroxylase
D. * phenylalanine hydroxylase
E. None of the above
148.
The rate-limiting step in the biosynthesis of norepinephrine involves the enzyme:
A. tryptophan hydroxylase
B. phenylalanine hydroxylase
C. dopamine beta-hydroxylase
D. * tyrosine hydroxylase
E. tryptophan decarboxylase
149.
The single best indicator of opioid drug addiction (psychological dependence) is:
A. An increasing need for the drug over time
B. Complaint of pain exceeding that expected for a given medical problem
C. Development of a withdrawal syndrome when the drug is stopped
D. * Evidence of adverse life consequences from drug use
E. Requesting a specific opioid by name
150.
The Syndrome of Imminent Death, also known as “actively dying”, typically
includes which one of the following constellation of findings:
A. Depression, cool extremities and pooled oropharyngeal secretions
B. Depression, warm extremities and polyuria
C. * Delirium, cool extremities, and pooled oropharyngeal secretions
D. Delirium, warm extremities and polyuria
E. Delirium, warm extremities and pooled oropharyngeal secretions
151.
The use of methadone in the treatment of heroin addiction continues to be
controversial. It would therefore be advantageous to find a substance with the beneficial
effects of methadone, but without its undesirable characteristics. The correct statement is
that:
A. methadone is not physically addicting and therefore very useful in treating heroin
addicts
B. the withdrawal syndrome of methadone is of shorter duration than that of heroin
C. * propoxyphene may successfully suppress the withdrawal syndrome in heroinaddicted individuals
D. although propoxyphene does block heroin withdrawal, it itself is not physically
addicting
E. propoxyphene may be successfully substituted for heroin in the addicted individual
and abruptly discontinued after three to four weeks without signs of an abstinence
syndrome
152.
This anesthetic has become obsolete due to its potential for causing nephrotoxicity.
Anesthetic Partition Minimum Alveolar Coefficient Concentration (MAC)
A. halothane
B. enflurane
C. nitrous oxide
D. isoflurane
E. * methoxyflurane
153.
TRUE statement concerning the pharmacologic effects of salicylates include:
salicylates are thought to exert their activity at least partially by inhibiting prostaglandin
synthetase; high-dose salicylate therapy (> 5 grams/day) lowers the serum uric acid
concentration; the effect of salicylates upon platelet aggregation is irreversible unlike that of
other nonsteroidal anti-inflammatory drugs; salicylate overdose is potentially fatal; however,
prompt administration of acetylcysteine will avert this danger
A. * salicylates are thought to exert their activity at least partially by inhibiting
prostaglandin synthetase, high-dose salicylate therapy (> 5 grams/day) lowers the
serum uric acid concentration, the effect of salicylates upon platelet aggregation is
irreversible unlike that of other nonsteroidal anti-inflammatory drugs
B. salicylates are thought to exert their activity at least partially by inhibiting
prostaglandin synthetase, the effect of salicylates upon platelet aggregation is
irreversible unlike that of other nonsteroidal anti-inflammatory drugs
C. high-dose salicylate therapy (> 5 grams/day) lowers the serum uric acid
concentration only
D. salicylates are thought to exert their activity at least partially by inhibiting
prostaglandin synthetase only
E. All of the above
154.
TRUE statement regarding midazolam as an adjunct to anesthesia:
A. provides analgesia
B. provides adequate skeletal muscle relaxation alone
C. * useful for ameliorating seizure activity
D. produces cardiovascular stimulation
E. contraindicated in patients with asthma or COPD
155.
True statement regarding thiopental:
A. causes increased EEG activity
B.
C.
D.
E.
contraindicated in a patient with epilepsy
provides analgesia
* associated with dose-dependent respiratory depression
provides adequate skeletal muscle relaxation alone
156.
True statements concerning the parasympathetic and sympathetic branches of the
autonomic nervous system include:
A. parasympathetic branch predominates under normal physiologic conditions while the
sympathetic branch predominates during stressful episodes
B. acetylcholine is the chemical mediator at the ganglia of both branches and causes
stimulation of nicotinic receptors postsynaptically
C. the adrenal medulla acts as a postganglionic sympathetic neuron and produces
norepinephrine which travels via the bloodstream to stimulate alpha, beta-1 and beta1 receptors
D. * parasympathetic branch predominates under normal physiologic conditions while
the sympathetic branch predominates during stressful episodes & acetylcholine is the
chemical mediator at the ganglia of both branches and causes stimulation of nicotinic
receptors postsynaptically
E. parasympathetic branch predominates under normal physiologic conditions while the
sympathetic branch predominates during stressful episodes & the adrenal medulla
acts as a postganglionic sympathetic neuron and produces norepinephrine which
travels via the bloodstream to stimulate alpha, beta-1 and beta-1 receptors
157. TRUE statements concerning the pharmacologic properties of salicylates include: high
dose aspirin therapy can lower the serum urate concentration; aspirin is metabolized by a
combination of 1st and zero order kinetics; aspirin exerts its actions primarily by
inhibition of cyclooxygenase; aspirin overdose causes significant hepatic toxicity if
ingested in sufficient quantities
A. * high dose aspirin therapy can lower the serum urate concentration; aspirin is
metabolized by a combination of 1st and zero order kinetics; aspirin exerts its
actions primarily by inhibition of cyclooxygenase
B. high dose aspirin therapy can lower the serum urate concentration; aspirin exerts
its actions primarily by inhibition of cyclooxygenase
C. aspirin is metabolized by a combination of 1st and zero order kinetics, aspirin
overdose causes significant hepatic toxicity if ingested in sufficient quantities
D. aspirin overdose causes significant hepatic toxicity if ingested in sufficient
quantities only
E. All of the above
158.
When using patient controlled analgesia, the lockout interval for a bolus (prn) dose
should be set between:
A. * 5-30 minutes
B. 30-60 minutes
C. 60-90 minutes
D. 90-120 minutes
E. 120-180 minutes
159.
Which of the following effects would you expect to see if you initially administer
atropine followed by the alpha agonist? Mydriasis; bradycardia; urinary retention; salivation
A. mydriasis; bradycardia; urinary retention
B. * mydriasis; urinary retention
C. bradycardia, salivation
D. salivation only
E. All of the above
160.
Which of the following is a characteristic of a patient who is in a persistent
vegetative state?:
A. The patient has no corneal, gag, or cough reflexes.
B. The patient has the ability to recognize family and friends
C. The patient is clinically brain dead
D. * The patient is in a coma at first, but then recovers sleep-wake cycles
E. There is a 5% chance that the patient will recover higher cognitive function
161.
Which of the following possess(es) some antagonist activity at opioid receptors
naloxone; pentazocine; butorphanol; nalorphine
A. Naloxone; Pentazocine; butorphanol
B. pentazocine; nalorphine
C. nalorphine only
D. * All of the above
E. Butorphanol only
162.
Which of the following regulatory agents or conditions would be expected to cause a
decrease in the number of beta-adrenergic receptors? Propranolol; guanethidine; alcohol
withdrawal; beta-adrenergic agonists
A. propranolol, guanethidine, alcohol withdrawal
B. propranolol, alcohol withdrawal
C. . guanethidine, alcohol withdrawal,. beta-adrenergic agonists
D. beta-adrenergic agonists only
E. * All of the above
163.
Which of the following statements are true? terbutaline is relatively specific for
Beta-2 receptors and is used as a bronchodilator; guanethidine blocks responses to indirect
acting sympathomimetic amines and potentiates they actions of exogenous epinephrine by
inhibiting neuronal uptake; MAO (monoamine oxidase) is the enzyme which catalyzes the
oxidative deamination of catecholamines to form aldehyde derivatives; yohimbine is an
ergot alkaloid which has beta blocking activity
A. * terbutaline is relatively specific for Beta-2 receptors and is used as a
bronchodilator; guanethidine blocks responses to indirect acting sympathomimetic
amines and potentiates they actions of exogenous epinephrine by inhibiting neuronal
uptake; MAO (monoamine oxidase) is the enzyme which catalyzes the oxidative
deamination of catecholamines to form aldehyde derivatives
B. terbutaline is relatively specific for Beta-2 receptors and is used as a bronchodilator;
MAO (monoamine oxidase) is the enzyme which catalyzes the oxidative
deamination of catecholamines to form aldehyde derivatives
C. guanethidine blocks responses to indirect acting sympathomimetic amines and
potentiates they actions of exogenous epinephrine by inhibiting neuronal uptake only
D. terbutaline is relatively specific for Beta-2 receptors and is used as a bronchodilator
only
E. All of the above
164.
Which of the following statements concerning the intravenous administration of
catecholamines is (are) true? isoproterenol causes a decrease in peripheral resistance and A
significant increase in pulse rate; norepinephrine causes an increase in pulse rate and
peripheral resistance; epinephrine causes an increase in pulse rate and a decrease in
peripheral resistance; norepinephrine causes an increase in systolic blood pressure and a
decrease in diastolic pressure
A. isoproterenol causes a decrease in peripheral resistance and a significant increase in
pulse rate; norepinephrine causes an increase in pulse rate and peripheral resistance;.
epinephrine causes an increase in pulse rate and a decrease in peripheral resistance
B. * isoproterenol causes a decrease in peripheral resistance and a significant increase
in pulse rate; epinephrine causes an increase in pulse rate and a decrease in
peripheral resistance
C. norepinephrine causes an increase in pulse rate and peripheral resistance only
D. epinephrine causes an increase in pulse rate and a decrease in peripheral resistance
only
E. All of the above
165.
Which of the following statements is (are) true? the veins are predominantly under
sympathetic control; the heart is predominantly under sympathetic control; the urinary
bladder is predominantly under parasympathetic control; the arterioles are predominantly
under parasympathetic control
A. the veins are predominantly under sympathetic control; the heart is predominantly
under sympathetic control; the urinary bladder is predominantly under
parasympathetic control
B. * the veins are predominantly under sympathetic control; the urinary bladder is
predominantly under parasympathetic control
C. the heart is predominantly under sympathetic control; the arterioles are
predominantly under parasympathetic control
D. the arterioles are predominantly under parasympathetic control only
E. All are correct
166.
Which one of the following statements about depression at end-of-life is true:
A. Clinical depression is a normal stage of the dying process
B. Depression associated with HIV is more difficult to treat than in cancer patients
C. * Feelings of hopelessness/worthlessness are indicators of a clinical depression
D. The degree of appetite and sleep disturbance is predictive of response to antidepressant medication
E. Tricyclic antidepressants are the first choice for drug therapy
167.
Which one of the following statements concerning nausea while taking opioids is
true:
A. Nausea to opioids is due to bowel distention and stimulation of the vagus nerve
B. Nausea to opioids is due to decreased bowel motility
C. Nausea to opioids is usually accompanied with itching
D. Nausea to opioids represents a drug allergy
E. * Nausea to opioids resolves in most patients within 7 days
168.
Your pharmacology laboratory has been given a new compound for screening. It is
found to increase the heart rate but has no effect on the eye. It also caused a significant
reduction in diastolic blood pressure as well as relaxation of the bronchial muscle. This
compound would be categorized as:
A. antimuscarinic
B. * beta agonist
C. beta antagonist
D. alpha antagonist
E. muscarinic
169.
Your pharmacology laboratory has been given an unknown compound for screening.
It is found to contract the sphincter muscle of the eye, decrease the heart rate, increase
bronchial gland secretions and causes sweating. You would classify this compound as (an):
A. antimuscarinic
B. alpha agonist
C. alpha antagonist
D. beta antagonist
E. * muscarinic
170.
Your pharmacology laboratory has received a compound for testing purposes. Initial
experimentation with the compound yields the following observations: It causes contraction
of the radial muscle of the iris, constriction of the arterioles of the skin and mucosa, and
hyperglycemia. You would classify this compound as a(an):
A. anticholinergic
B. beta blocker
C. * alpha agonist
D. muscarinic
E. antimuscarinic
171.
Your pharmacology laboratory has received a new compound for screening. Initial
experimentation with the compound yields the following findings: It causes contraction of
the detrusor muscle of the urinary bladder, contraction of the sphincter muscle of the iris,
and secretion of lacrimal glands. With this information you might initially classify the
compound as a(an):
A. alpha agonist
B. * muscarinic agonist
C. beta agonist
D. anticholinergic
E. alpha antagonist
172.
Which of the following antitubercular agents is associated with the development of
ototoxicity?
A. rifampin
B. ethambutol
C. isoniazid
D. * streptomycin
E. para-aminosalicylic acid
173.
. Which of the following antibiotics is most closely associated with the development
of hepatitis?
A. kanamycin
B. penicillin G
C. tetracycline
D. * isoniazid
E. ethambutol
174.
10-30 years postexposure - cardiac and neurologic destruction
A. Syphilis
B. Syphilis Primary
C. * Syphilis Teritiary
D. Syphilis Secondary
E. none of the above
175.
A 23 years old patient is pregnant with a history of severe (anaphylactic) penicillin
allergy. To prepare her for an upcoming tooth extraction you prescribe an antimicrobial
medication that can be taken prophlaxis 2 hrs prior to the procedure. This medication is
A. Cefaclor
B. Doxycycline
C. Erythromycin base
D. * Erythromycin stearate
E. Gentamycin
176.
A bacteriostatic antimicrobial that inhibits protein synthesis by blocking attachment
of aminoacyi t-RNA to the A site on the 30S bacterial ribosome
A. Aminoglycosides
B. Fluoroquinolones
C. Metronidazole
D. Sulfonamide
E. * Tetracycline
177.
All of the following are considered primary drugs in the current therapeutic
approaches to the treatment of tuberculosis EXCEPT:
A. streptomycin
B. * para-aminosalicylic acid
C. isoniazid
D. rifampin
E. ethambutol
178.
Alternative antimicrobial drug(s) for treating penicillin-resistant S. aureus:
A. vancomycin (Vancocin)
B.
C.
D.
E.
nafcillin (Nafcil, Unipen)
* methicillin (Staphcillin)
* All of the above
none of the above
179.
Aminoglycosides antibacterial
A. hetacillin
B. aztreonam
C. * gentamicin (Garamycin)
D. vancomycin (Vancocin)
E. all of the above
180.
Aminoglycosides are: excreted renally; bactericidal; potentially toxic to the auditory
and vestibular portions of the eighth cranial nerve; potentially nephrotoxic
A. excreted renally; bactericidal; potentially toxic to the auditory and vestibular
portions of the eighth cranial nerve
B. excreted renally; potentially toxic to the auditory and vestibular portions of the
eighth cranial nerve
C. Bactericidal; potentially toxic to the auditory and vestibular portions of the eighth
cranial nerve
D. * All of the above
E. only potentially nephrotoxic
181.
Aminoglycosides: must be dosed on the basis of renal function; are primarily active
against gram-negative aerobes; are not active against anaerobes; penicillin may be
synergistic against enterococcus
A. must be dosed on the basis of renal function; are primarily active against gramnegative aerobes; are not active against anaerobes
B. must be dosed on the basis of renal function; are not active against anaerobes
C. primarily active against gram-negative aerobes, penicillin may be synergistic against
enterococcus
D. penicillin may be synergistic against enterococcus only
E. * All of the above
182.
Aminoglycosides: are very popular; are well absorbed from the gastrointestinal tract;
do not penetrate pulmonary secretions well; do not appear in urine in high concentrations
A. are very popular; are well absorbed from the gastrointestinal tract; do not penetrate
pulmonary secretions well
B. * are very popular; do not penetrate pulmonary secretions well
C. are well absorbed from the gastrointestinal tract only
D. only are very popular
E. All of the above
183.
An antitubercular agent which is associated with the development of ocular toxicity
is:
A. rifampin
B. * ethambutol
C. isoniazid
D. streptomycin
E. para-aminosalicylic acid
184.
Antibacterials regarded as generally safe to prescribe in pregnancy.
A. erythromycins
B. cephalosporins
C. tetracyclines
D. * erythromycins & cephalosporins
E. none of the above
185.
Antibiotic's clearance from the body most likely influenced by severe hepatic
disease:
A. penicillins
B.
C.
D.
E.
clindamycin (Cleocin)
rifampin (Rimactane)
* clindamycin & rifampin
none of the above
186.
Associated with "gray baby syndrome" which is characterized by pallor, cyanosis,
and even death.
A. tetracyclines
B. * chloramphenicol (Chloromycetin)
C. third-generation cephalosporins
D. All of the above
E. none of the above
187.
Azole most likely to get into the cerebral spinal fluid (CSF):
A. ketoconazole
B. itraconazole
C. * fluconazole
D. all of the above
E. none of the above
188.
Bacterial resistance to this drug results from a plasmid mediated mechanism that
results in an altered ribosomal binding site within the bacteria
A. Ampicillin
B. * Azithromycin
C. Penicillin
D. Piperacillin
E. Vancomycin
189.
Bactericidal drugs that act by inhibiting DNA gyrase
A. Aminoglycoside
B. * Fluoroquinolones
C. Macrolides
D. Rifamycins
E. Sulfonamides
190.
Beta-lactamase inhibitor:
A. vancomycin (Vancocin)
B. sulbactam
C. clavulanate
D. * sulbactam & clavulanate
E. none of the above
191.
Binds to specific receptors located in the bacterial cytoplasmic cell membrane and
inhibits transpeptidase enzymes, preventing cross linking of peptidoglycan chains in the cell
wall
A. * Cephalosporins
B. Daptomycin
C. Linezolid
D. Metronidazole
E. Rifampin
192.
Binds to the 30s bacterial ribosome and blocks the initiation step in protein synthesis.
May cause miscoding
A. * Aminoglycosides
B. Daptomysin
C. Linezolid
D. Metronidazole
E. Rifampin
193.
Binds to the 50 s bacterial ribosome and prevents the translocation step in protein
synthesis
A. Aminoglycosides
B.
C.
D.
E.
Daptomycin
Linezolid
* Macrolides
Metronidazole
194.
Binds to the D-Ala-D-Ala terminal end of peptidoglycan side chains, resulting in
inhibition of transglycosylation and disruption of cross-linking in the cell wall
A. Chloramphenicol
B. Clavulanic
C. Imipenem
D. * Vancomycin
E. Metronidazole
195.
Cavalier use of antibiotics in the hospital setting may be associated with: increased
cost; increased toxicity; alteration of hospital flora; increased incidence of infections
A. increased cost; increased toxicity; alteration of hospital flora
B. increased cost; alteration of hospital flora
C. increased toxicity
D. increased incidence of infections
E. * All of the above
196.
Cefazolin is generally preferred among the first-generation cephalosporins, because
it: has a favorable pharmacokinetic profile; has a longer half-life than comparable drugs;
may be dosed every 8 hours; costs less for a hospital to utilize, since it may be given less
frequently than comparable drugs
A. has a favorable pharmacokinetic profile; has a longer half-life than comparable
drugs; may be dosed every 8 hours
B. has a favorable pharmacokinetic profile; may be dosed every 8 hours
C. has a longer half-life than comparable drugs; costs less for a hospital to utilize, since
it may be given less frequently than comparable drugs
D. costs less for a hospital to utilize, since it may be given less frequently than
comparable drugs only
E. * All of the above
197. Cefazolin is generally preferred among the first-generation cephalosporins, because it:
has a favorable pharmacokinetic profile;
has a longer half-life than comparable drugs;
may be dosed every 8 hours;
costs less for a hospital to utilize, since it may be given less
frequently than comparable drugs
A. has a favorable pharmacokinetic profile; has a longer half-life than comparable
drugs; may be dosed every 8 hours
B. has a favorable pharmacokinetic profile; may be dosed every 8 hours
C. 2, 4 has a longer half-life than comparable drugs only
D. has a favorable pharmacokinetic profile only
E. * All of the above
198.
Chancre, highly infectious
A. * Syphilis Primary
B. Syphilis
C. Syphilis Teritiary
D. Syphilis Secondary
E. none of the above
199.
Chloramphenicol is associated with: pancytopenia; gray-baby syndrome; reversible
erythroid-suppression of bone marrow; renal failure
A. * Pancytopenia; gray-baby syndrome;reversible erythroid-suppression of bone
marrow
B. Pancytopenia; reversible erythroid-suppression of bone marrow
C. gray-baby syndrome, renal failure
D. renal failure only
E. All of the above
200. Chloramphenicol is associated with:
Pancytopenia;
gray-baby syndrome;
reversible erythroid-suppression of bone marrow;
renal failure;
A. * pancytopenia; gray-baby syndrome; reversible erythroid-suppression of bone
marrow
B. pancytopenia; reversible erythroid-suppression of bone marrow
C. gray-baby syndrome; renal failure
D. renal failure only
E. All of the above
201.
Circumstances associated with allergy to penicillins may include: maculopapular or
urticarial rash; anaphylaxis; anaphylactic reaction to penicillin skin-tests; exposure to
penicillins in food
A. maculopapular or urticarial rash; anaphylaxis; anaphylactic reaction to penicillin
skin-tests
B. maculopapular or urticarial rash; anaphylactic reaction to penicillin skin-tests
C. anaphylaxis; exposure to penicillins in food
D. exposure to penicillins in food only
E. * All of the above
202.
Clindamycin is: associated with diarrhea; active against anaerobes; active against
some gram-positive organisms; active against some gram-negative organisms
A. * associated with diarrhea; active against anaerobes; active against some grampositive organisms
B. associated with diarrhea; active against some gram-positive organisms
C. active against anaerobes only
D. only active against some gram-negative organisms
E. All of the above
203. Clindamycin is:
associated with diarrhea;
active against anaerobes;
active against some gram-positive organisms;
active against some gram-negative organisms
A. * associated with diarrhea; active against anaerobes; active against some grampositive organisms
B. associated with diarrhea; active against some gram-positive organisms
C. active against anaerobes; active against some gram-negative organisms
D. active against some gram-negative organisms only
E. All of the above
204.
Combination of metronidazole (Flagyl) and penicillin in treating an abscess caused
by beta-lactamase producing Bacteriodes and anaerobic streptococci is an example of:
A. synergistic drug treatment
B. antagonistic drug effects
C. * additive drug effects
D. all of the above
E. none of the above
205.
Combination of these two drugs would produce an antagonistic (undesirable)
antimicrobial effect when treating an infection
A. Amoxicillin+ gentamicin
B. Cephalexin+ gentamicin
C. Sulfamethoxazole+ trimethoprim
D. * Tetracycline+ amoxicillin
E. Vancomycin+ gentamicin
Diagnoised with tissue culture, MictoTak, chlamydiazyme
A. Candidias
B. Syphllis
C. Chalmdyia
D. * Chalymida
E. none of the above
207.
Diagnoised with viral culture
A. Syphilis
B. Chalymida
C. * Candidias
D. Chalmdyia
E. none of the above
208.
Failure of first-line drugs in the initial treatment of apparent pulmonary tuberculosis
may have several causes including poor patient compliance and atypical mycobacterial
infections, the latter usually requiring use of second-line drugs. All of the following are firstline drugs EXCEPT:
A. streptomycin
B. rifampin
C. * ethionamide
D. ethambutol
E. isoniazid
209.
FALSE statement about penicillin G
A. treatment of choice for viridans group streptococcal endocarditis.
B. pen G and Pen V are the two natural penicillins
C. pen G can be combined with procaine, extending drug half-life
D. * pen G most effective when given orally
E. none of the above
210.
Females: odorless, white or yellow cheesy discharge with itching
A. Chalymida
B. Syphilis
C. Chalmdyia
D. * Candidias
E. none of the above
211.
Given by i.v. administration, the drug of choice for nearly all life-threatening
mycotic infections--usually used as the initial induction regimen
A. ketoconazole
B. itraconazole
C. flucytosine
D. * amphotericin B
E. nystatin
212.
Imipenem is a beta-lactam antibiotic which is neither a penicillin nor a
cephalosporin. Correct statements regarding imipenem include: it covers an extremely broad
spectrum of microorganisms; it is very active against many gram negative rods; resistant
pseudomonas may emerge during treatment; it should not be given to patients having a
history of allergic reactions to penicillin
A. * it covers an extremely broad spectrum of microorganisms; it is very active against
many gram negative rods; resistant pseudomonas may emerge during treatment
B. it covers an extremely broad spectrum of microorganisms; resistant pseudomonas
may emerge during treatment
C. it is very active against many gram negative rods; it should not be given to patients
having a history of allergic reactions to penicillin
D. it should not be given to patients having a history of allergic reactions to penicillin
only
206.
E. All of the above
213.
Imipenem is a beta-lactam antibiotic which is neither a penicillin nor a
cephalosporin. Correct statements regarding imipenem include: it covers an extremely broad
spectrum of microorganisms; it is very active against many gramnegative rods; resistant
pseudomonas may emerge during treatment; it should not be given to patients having a
history of allergic reactions to penicillin
A. it covers an extremely broad spectrum of microorganisms; it is very active against
many gramnegative rods; resistant pseudomonas may emerge during treatment
B. it covers an extremely broad spectrum of microorganisms; resistant pseudomonas
may emerge during treatment
C. it is very active against many gramnegative rods; it should not be given to patients
having a history of allergic reactions to penicillin
D. only it should not be given to patients having a history of allergic reactions to
penicillin
E. * All of the above
214.
Impaired vision is an adverse effect of:
A. carbenicillin
B. * ethambutol
C. rifampin
D. colistin
E. cycloserine
215.
Inhibition of fungal squalene epoxidase is this antifungal drug's mechanism of action:
A. ketoconazole
B. fluconazole
C. * terbinafine
D. nystatin
E. all of the above
216.
Inhibits bacterial cell wall synthesis:
A. streptomycin
B. * vancomycin (Vancocin)
C. doxycycline (Vibramycin, Doryx)
D. gentamicin (Garamycin)
E. none of the above
217.
Inhibits the formation of folic acid by competing with PABA for dihydropteroate
synthase
A. Metronidazole
B. Rifampin
C. * Sulfamethoxazole
D. Trimethoprin
E. Linezolid
218.
Isoniazid-induced liver damage:
A. occurs primarily in patients under 30 years of age
B. . occurs with increased frequency in patients receiving concomitant
C. * is probably due to the formation of a toxic hydrazine metabolite that
D. is frequently associated with allergic manifestations such as
E. All of the above
219.
Macrolides and tetracyclines are bacteriostatic while aminoglycosides are
bactericidal. This difference results from some reasons
A. Aminoglycosides binding to ribosomes is stronger almist irreversible
B. Aminoglycosides can cause misreading of mRNA, resulting in build-up of toxic
proteins
C. Aminoglycosides cause DNA damage. While tetracycline and macrolides do not
D. Bacteria develop a rapid tolerance to tetracyclines and macrolides
E. * A and B
220.
Mechanism of action: amoxicillin (Amoxil Polymox)
A. inhibits bacterial cell wal biosynthesis
B. interferes with transpeptidation preventing peptidoglycan chain crosslinking
C. * inhibits bacterial cell wal biosynthesis & interferes with transpeptidation
preventing peptidoglycan chain crosslinking
D. inhibits blood cell
E. none of the above
221.
More likely to cause polyneuritis in patients who are"slow acelators"; this antibiotic
is inactivated by the liver.
A. clindamycin (Cleocin)
B. * isoniazid (INH)
C. nafcillin (Nafcil, Unipen)
D. vancomycin (Vancocin)
E. none of the above
222.
More serious adverse effects: itraconazole (azole) or flucytosine (pyrimidine
analogue)
A. * flucytosine
B. itraconazole
C. clot blood
D. all of the above
E. none of the above
223.
Most adults who are at high risk of developing pulmonary tuberculosis should be
treated with:
A. isoniazid 100 mg/day for 6 months
B. * isoniazid 300 mg/day for 12 months
C. ethambutol 400 mg/day for 12 months
D. rifampin 600 mg/day for 24 months
E. No treatment is necessary
224.
Most potent of the presently available anti-fungal azoles:
A. fluconazole
B. * itraconazole
C. ketoconazole
D. all of the above
E. none of the above
225.
Mr.Antee Biatek, a 30 yr old woman is scheduled to undergo an exploration
laparotomy to confirm the diagnosis of an ectopic pregnancy. She has a history of several
penicillin allergy. For prophylaxis prior to her surgery you administer
A. * Chloramphenicol
B. Erythromycin stearate
C. Gentamycin + vancomycin i.v
D. Rifampin
E. Streptogramins A+B (synercid)
226.
Of ketoconazole, itraconazole, and fluconazole--most likely to cause increased
arrhythmogenic effects of astemizole or terphidine
A. * ketoconazole
B. itraconazole
C. fluconazole
D. all of the above
E. none of the above
227.
Ototoxicity and nephrotoxicity associated with this antibiotic:
A. cefotaxime (Claforan)
B. * amikacin (Amikin)
C. aztreonam (Azactan)
D. ceftriaxone (Rocephin)
E. All of the above
228. Patients having a history of a severe, immediate reaction to penicillin:
may be given a cephalosporin without concern;
have a definite risk of reaction to any cephalosporin;
have a low risk of having a reaction to a broad spectrum antipseudomonal penicillin;
have a high risk of hypersensitivity to a broad spectrum anti- pseudomonal penicillin;
A. may be given a cephalosporin without concern; have a definite risk of reaction to any
cephalosporin; have a low risk of having a reaction to a broad spectrum
antipseudomonal penicillin
B. may be given a cephalosporin without concern; have; have a low risk of having a
reaction to a broad spectrum antipseudomonal penicillin
C. * have a definite risk of reaction to any cephalosporin; have a high risk of
hypersensitivity to a broad spectrum anti- pseudomonal penicillin
D. have a high risk of hypersensitivity to a broad spectrum anti- pseudomonal penicillin
only
E. All of the above
229.
Penicillin: has a half-life of about 30 minutes; is primarily excreted renally; has an
average duration of about 26 days when administered in the benzathine form; readily enters
CSF when meninges are normal
A. * has a half-life of about 30 minutes; is primarily excreted renally; has an average
duration of about 26 days when administered in the benzathine form
B. has a half-life of about 30 minutes; has an average duration of about 26 days when
administered in the benzathine form
C. is primarily excreted renally, readily enters CSF when meninges are normal
D. readily enters CSF when meninges are normal only
E. All of the above
230.
Properties of tetracycline (Achromycin):
A. inhibitor of bacterial cell wall synthesis
B. * dug of choice in treating typhus
C. cleared primarily by the liver
D. inhibitor of bacterial cell wall synthesis & dug of choice in treating typhus
E. none of the above
231.
Prophylactic antibiotics in the United States are associated with: increased incidence
of infections in some cases; decreased incidence of infections in some cases; increased cost
in some cases; decreased cost in some cases
A. increased incidence of infections in some cases; decreased incidence of infections in
some cases; increased cost in some cases
B. increased incidence of infections in some cases; increased cost in some cases
C. increased cost in some cases only
D. decreased cost in some cases only
E. * All of the above
232.
Rifampicine is used mainly in the treatment of :
A. Cholera
B. Typhoid fever
C. * Tuberculosis
D. Rickettsial diseases
E. Pseudomonas infections
233.
Rifampin is used mainly in the treatment of:
A. cholera
B. typhoid fever
C. * tuberculosis
D. rickettsial diseases
E. pseudomonas infections
234.
Some potential barriers to successful treatment of an abscess-cavitary infection with
an aminoglycoside include: inactivation of aminoglycoside by an acidic medium; binding of
the drug by pus; decreased ability of the drug to penetrate the abscess because of poor
vascularity; inactivation of aminoglycoside by alkaline medium
A. * inactivation of aminoglycoside by an acidic medium; binding of the drug by pus;
decreased ability of the drug to penetrate the abscess because of poor vascularity
B. . inactivation of aminoglycoside by an acidic medium; decreased ability of the drug
to penetrate the abscess because of poor vascularity
C. binding of the drug by pus & inactivation of aminoglycoside by alkaline medium
D. only inactivation of aminoglycoside by alkaline medium
E. All of the above
235.
Special concerns in using antimicrobials in newborns and young children:
A. sulfonamides can be safely given to newborns
B. tetracyclines should not be administered to children betlow the age of eight
C. newborns should not be given chloramphenical because they are unable to
metabolize the drug adequately.
D. * tetracyclines should not be administered to children betlow the age of eight &
newborns should not be given chloramphenical because they are unable to
metabolize the drug adequately.
E. none of the above
236.
Suitable for treatment of bacterial meningitis caused by H. influenza:
A. * cefotaxime (Claforan)
B. cephalexin (Keflex)
C. cephalothin (Keflin)
D. cefadroxil (Duricef, Ultracef)
E. none of the above
237.
Sulfonamides: are competitive antagonists of PABA, and thereby decrease bacterial
utilization of para-aminobenzoic acid in the synthesis of folic acid; do not affect mammalian
cells, since they require pre-formed folic acid; are synergistic with trimethoprim, which
inhibits dihydrofolate reductase (which reduces dihydrofolate to tetrahydrofolate); can be
antagonized by PABA
A. are competitive antagonists of PABA, and thereby decrease bacterial utilization of
para-aminobenzoic acid in the synthesis of folic acid; do not affect mammalian cells,
since they require pre-formed folic acid; are synergistic with trimethoprim, which
inhibits dihydrofolate reductase (which reduces dihydrofolate to tetrahydrofolate)
B. A,C are competitive antagonists of PABA, and thereby decrease bacterial utilization
of para-aminobenzoic acid in the synthesis of folic acid; are synergistic with
trimethoprim, which inhibits dihydrofolate reductase (which reduces dihydrofolate to
tetrahydrofolate)
C. do not affect mammalian cells, since they require pre-formed folic acid & can be
antagonized by PABA
D. can be antagonized by PABA only
E. * All of the above
238.
Synergistic antimicrobial combination:
A. amoxicillin-clavulanate
B. ampicillin -streptomycin
C. sulbactam-ampicillin
D. * all of the above
E. none of the above
239.
Tetracyclines: are bacteriostatic in vitro; are bactericidal in vitro; are effective
against rickettsiae; interfere primarily with cell wall synthesis
A. are bacteriostatic in vitro; are bactericidal in vitro; are effective against rickettsiae
B. * are bacteriostatic in vitro; are effective against rickettsiae
C. are bactericidal in vitro; interfere primarily with cell wall synthesis
D. interfere primarily with cell wall synthesis only
E. All of the above
240. The structure is administered along with a penicillin in a preparation known as Tazocin or
Zosyn. Which penicillin is used?
A. Amoxicillin
B. * Piperacillin
C. Ampicillin
D. Methicillin
E. None of the above
241.
The combination of sulfamethoxazole and trimethoprim is a useful drug for treating
UTIs. Bacterial resistance to trimethoprim occurs primarily by
A. Acetylation of trimethoprim
B. Acetylation of the drug receptor
C. Decreased bacterial uptake
D. * Upregulation of dihydrofolate reductase
E. Upregulation of beta-lactamase
242.
The drug of choice for methicillin-resistant Staphylococcus aureus is:
A. oxacillin
B. gentamicin
C. * vancomycin
D. piperacillin
E. None of the above
243. The drug of choice for methicillin-resistant Staphylococcus aureus is:
A. oxacillin
B. gentamicin
C. * vancomycin
D. piperacillin
E. None of the above
244.
The drug of choice for Streptococcus pneumonia (pneumococcus) in a patient with
no drug allergies is:
A. erythromycin
B. * penicillin
C. vancomycin
D. ceftriaxone
E. gentamicin
245. The following structure (methicillin) was an important penicillin that was introduced in
the nineteen sixties to counter the threat of penicillin resistant strains of S. Aureus. Which of
the following statements is true regarding the above structure?
A. * There is no electron withdrawing group on the side chain, and so it is acid
sensitive.
B. It can be taken orally
C. It is more active than penicillin G
D. It has a broader spectrum of activity compared to penicillin G.
E. None of the above
246. The following structure (ritonavir) is a protease inhibitor. What feature of the protease
enzyme was the inspiration for the design of this particular structure?
A. The presence of a flap region over the active site
B. The presence of isoleucine residues in the flap region
C. The presence of aspartyl groups in the active site
D. * The symmetrical nature of the active site
E. All of the above
247.
The following structure is a protease inhibitor. What is the name of the structure?
A. * Ritonavir
B. Lopinavir
C. Saquinavir
D. Aciclovir
E. all of the above
248. The following structure is a synthetic antibacterial agent called ciprofloxacin. What is the
most likely mechanism of bacterial resistance to the above structure?
A. Modified target enzyme
B. Enzymatic modification of the structure
C. * Efflux mechanisms
D. Increased production of target enzymes
E. All of the above
249. The HIV protease enzyme is symmetrical, whereas mammalian proteases are not. What
significance might this have?
A. Inhibitors of the HIV protease enzyme must also be symmetrical.
B. * It may be possible to design inhibitors that prove selective for the HIV protease
over mammalian proteases.
C. Only symmetrical substrates are cleaved by HIV protease.
D. There is no significance
E. All of the above
250.
The route by which isoniazid is usually administered is:
A. * oral
B. intramuscular injection of solution
C. intramuscular injection of suspension
D. subcutaneous injection of solution
E. intravenous
251.
This antibiotic may cause hemolysis if prescribed to a patient with glucose-6phosphate dehydrogenase deficiency:
A. penicillin V (Pen-Vee K, Veetids)
B. neomycin
C. Augmentin ( amoxicillin (Amoxil Polymox) and clavulanic acid)
D. * sulfisoxazole (Gantrisin)
E. none of the above
252.
Topical allylamine(s) used for treating tinea cruris and tinea corporis:
A. terbinafine
B. naftifine
C. * terbinafine & naftifine
D. clot blood
E. none of the above
253.
Treat with Metronidazole (Flagyl)
A. Trichomanas vaginalis
B. Trichomoanas vaginlais
C. Tricomanas Vaginalis
D. * Trichomanas Vaginalis
E. none of the above
254. Trimethoprim is often administered alongside a sulfonamide as a preparation known as
co-trimoxazole. Which of the following sulfonamides is used?
A. * Sulfamethoxazole
B. Sulfathiazole
C. Sulfadoxine
D. Sulfadiazine
E. None of the above
255.
Tubercle bacilli exist in tuberculous patients in three pools extracellular,intracellular,
and necrotic caseum. The only drug which is bactericidal for all three is:
A. isoniazid
B. ethambutol
C. pyrazinamide
D. * rifampin
E. streptomycin
256.
Vancomycin: is a treatment for clostridium difficile-induced diarrhea; is useful in
treating methicillin-resistant staphylococci; is potentially ototoxic; requires a dosage
adjustment when administered to patients with impaired renal function
A. is a treatment for clostridium difficile-induced diarrhea; is useful in treating
methicillin-resistant staphylococci; is potentially ototoxic
B. is a treatment for clostridium difficile-induced diarrhea; is potentially ototoxic
C. is useful in treating methicillin-resistant staphylococci; requires a dosage adjustment
when administered to patients with impaired renal function
D. requires a dosage adjustment when administered to patients with impaired renal
function only
E. * All of the above
257. Vancomycin:
is a treatment for clostridium difficile-induced diarrhea;
is useful in treating methicillin-resistant staphylococci;
is potentially ototoxic;
requires a dosage adjustment when administered to patients with impaired renal function;
A. is a treatment for clostridium difficile-induced diarrhea; is useful in treating
methicillin-resistant staphylococci; is potentially ototoxic
B. is a treatment for clostridium difficile-induced diarrhea; is potentially ototoxic
C. is useful in treating methicillin-resistant staphylococci; requires a dosage adjustment
when administered to patients with impaired renal function
D. requires a dosage adjustment when administered to patients with impaired renal
function only
E. * All of the above
258.
Various drugs may induce vitamin deficient states as an undesirable side effect.
Vitam B6 (pyridoxine) deficiency may be related to taking:
A. estrogen-containing oral contraceptives
B. colchicine
C. * isoniazid
D. All of the above
E. none of the above
259. What advantage does cidofovir have over aciclovir as an antiviral agent?
A. It has better oral bioavailability
B. It is less polar
C. * It has a broader spectrum of activity.
D. It has less side effects
E. All of the above
260. What crucial feature of a penicillin is involved in its mechanism of action?
A. Carboxylic acid
B. * beta-lactam ring
C. Acyl side chain
D. Thiazolidine ring
E. All of the above
261. What drugs are present in the preparation Augmentin?
A. Ticarcillin and clavulanic acid
B. Ampicillin and sulbactam
C. Ampicillin and clavulanic acid
D. * Amoxicillin and clavulanic acid
E. None of the above
262. What reaction is catalysed by a beta-lactamase enzyme?
A. The final cross linking reaction of a bacterial cell wall
B.
C.
D.
E.
The hydrolysis of the acyl side chain from penicillin structures
* The hydrolysis of the four membered ring present in penicillins
The biosynthesis of the penicillin structure from amino acids
None of the above
263.
What role does the acetoxy group at the 3-position of cephalosporins have in
enhancing antibacterial activity?
A. It acts as a steric shield and masks enzymatic attack at the beta-lactam ring.
B. * It acts as a good leaving group when the beta-lactam ring is opened
C. It takes part in a transesterification reaction with the carboxylic acid group at
position 4
D. It increases the reactivity of the beta-lactam ring by neighbouring group participation
E. None of the above
264.
What was the initial lead compound used in the design of saquinavir?
A. L-prolyl-L-phenylalanine
B. * L-phenylalanyl-L-proline
C. L-phenylalanyl-L-phenylalanine
D. L-prolyl-L-proline
E. None of the above
265.
What would be the effect of replacing the methoxy groups in methicillin with ethoxy
groups?
A. Activity would increase
B. * esistance to enzymatic hydrolysis would increase
C. Activity against a broad spectrum of bacterial strains would increase
D. Stability to acid catalysed hydrolysis would increase
E. None of the above
266.
Which form of resistance to the effect of antimicrobial drugs is specific to betalactam type antibiotics
A. Decreased drug uptake
B. Methylation of the antibiotic’s receptor
C. Decreased metabolic activation
D. Altered amount of drug receptor
E. * Increased enzymatic destruction of the antibiotic
267.
Which of the following agents used in the treatment of pulmonary tuberculosis has
the side effect of loss of perception of the color green?
A. isoniazid
B. rifampin
C. * ethambutol
D. streptomycin
E. para-amino salicylic acid
268.
Which of the following agents used in the treatment of pulmonary tuberculosis has a
toxicity of producing a lupus-like syndrome?
A. * isoniazid
B. rifampin
C. ethambutol
D. streptomycin
E. para-amino salicylic acid
269. Which of the following antibiotics is a tetracycline?
A. Chloramphenicol
B. * Doxycycline
C. Streptomycin
D. Erythromycin
E. None of the above
270.
Which of the following antibiotics is most closely associated with the development
of ocular toxicity?
A.
B.
C.
D.
E.
kanamycin
penicillin G
tetracycline
isoniazid
* ethambutol
271.
Which of the following antibiotics is most closely associated with the development
of renal and ototoxicity?
A. * kanamycin
B. penicillin G
C. tetracycline
D. isoniazid
E. ethambutol
272.
Which of the following antimicrobial drugs is bactericidal
A. Azithromycin
B. Doxycycline
C. Erythromycin
D. * Gentamicin
E. Tetracycyline
273.
Which of the following antinmicrobials is taken up into bacteria by an oxygendependent mechanim and is therefore ineffective against anaerobic organisms
A. Cefoxitin
B. Ticarcillin
C. Clindamycin
D. * Gentamicin
E. Metronidazole
274.
Which of the following antitubercular drugs has hyperuricemia as one of its welldocumented untoward effects?
A. streptomycin
B. isoniazid
C. cycloserine
D. rifampin
E. * pyrazinamide
275.
Which of the following drug combinations would produce a synergistic effect
A. Amoxicillin+ ampicillin
B. Cephalexin+ penicillin V
C. Sulfamethoxazole+ trimethoprim
D. Vancomycin+ gentamicin
E. * Sulfamethoxazole+ trimethoprim and Vancomycin+ gentamicin
276.
Which of the following drugs are generally considered to be bactericidal in vitro?
Cephalosporins; penicillins; aminoglycosides; clindamycin
A. * Cephalosporins; penicillins; aminoglycosides
B. cephalosporins & aminoglycosides
C. penicillins & clindamycin
D. clindamycin only
E. All of the above
277.
Which of the following is a bactericidal antitubercular drug that has an inhibitory
action on cell wall synthesis?
A. * isoniazid
B. rifampin
C. ethambutol
D. streptomycin
E. para-aminosalicylic acid
278.
Which of the following is an enzyme carried within the nucleocapsid of the flu virus?
A. Hemagglutinin
B. * RNA polymerase
C. DNA polymerase
D. Neuraminidase
E. All of the above
279. Which of the following is the general mechanism of action for erythromycin?
A. Inhibition of a metabolic enzyme
B. Inhibition of cell wall synthesis
C. * Disruption of protein synthesis
D. Inhibition of nucleic acid transcription and replication
E. None of the above
280. Which of the following is the general mechanism of action for fluoroquinolones?
A. Inhibition of a metabolic enzyme
B. Inhibition of cell wall synthesis
C. Disruption of protein synthesis
D. * Inhibition of nucleic acid transcription and replication
E. None of the above
281. Which of the following statements is accurate in explaining why Gram negative bacteria
are generally more resistant to penicillins than Gram positive bacteria?
A. Gram negative bacteria have a thicker cell wall
B. * Gram negative bacteria have an outer hydrophilic membrane that acts as an extra
barrier
C. Gram negative bacteria can concentrate beta-lactamase enzymes in the periplasmic
space
D. Gram negative bacteria produce smaller quantities of transpeptidase enzyme
E. None of the above
282. Which of the following statements is true regarding the properties of benzylpenicillin?
A. It is a bacteriostatic agent.
B. It is active over a wide range of bacterial species.
C. It is resistant to ?-lactamases.
D. * Certain individuals may have an allergic response to it.
E. All of the above
283.
The state of “general anesthesia” usually includes:
A. Analgesia
B. Loss of consciousness,
C. Inhibition of sensory and autonomic reflexes
D. Amnesia
E. *All of the above
284.
Indicate the anesthetic, which is an inhibitor of NMDA glutamate receptors:
A. Thiopental
B. Halothane
C. *Ketamine
D. Sevoflurane
E. None of the above
285.
An ideal anesthetic drug would:
A. Induces anesthesia smoothly and rapidly
B. Secure rapid recovery
C. Posses a wide margin of safety
D. Be devoid of adverse effects
E. *All of the above
286.
Which of the following general anesthetics belongs to inhalants?
A. Thiopental
B. *Desfluran
C. Ketamine
D. Propofol
E. Diprofol
Indicate the anesthetic, which is used intravenously:
A. *Propofol
B. Halothane
C. Desflurane
D. Nitrous oxide
E. All of the above
288.
Sevoflurane has largely replaced halothane and isoflurane as an inhalation anesthetic
of choice because:
A. Induction of anesthesia is achieved more rapidly and smoothly
B. Recovery is more rapid
C. It has low post- anesthetic organ toxicity
D. *All of the above
E. None of the above
289.
The limitation of sevoflurane is:
A. High incidence of coughing
B. Laryngospasm
C. *Chemically unstable
D. Centrally mediated sympathetic activation leading to a rise of BP and HR
E. Hepatotoxicity
290.
Which of the following inhalants lacks sufficient potency to produce surgical
anesthesia by itself and therefore is commonly used with another inhaled or intravenous
anesthetic?
A. Halothane
B. Sevoflurane
C. *Nitrous oxide
D. Desflurane
E. Halothane &Sevoflurane
291.
Which of the following inhaled anesthetics has rapid onset and recovery?
A. Nitrous oxide
B. Desflurane
C. Sevoflurane
D. *All of the above
E. None of the above
292.
Which of the following inhaled anesthetics is an induction agent of choice in patient
with airway problems?
A. Desfurane
B. Nitrous oxide
C. *Halothane
D. None of the above
E. All of the above
293.
Which of the following inhaled anesthetics can produce hepatic necrosis?
A. Soveflurane
B. Desflurane
C. *Halothane
D. Nitrous oxide
E. All of the above
294.
Indicated the inhaled anesthetic, which may cause nephrotoxicity:
A. Halothane
B. *Soveflurane
C. Nitrous oxide
D. Diethyl ether
E. None of the above
287.
295.
Which of the following inhaled anesthetics decreases metheonine synthase activity
and causes megaloblastic anemia?
A. Desflurane
B. Halothane
C. *Nitrous oxide
D. Soveflurane
E. None of the above
296.
Unlike inhaled anesthetics, intravenous agents such as thiopental, etomidate, and
propofol:
A. Have a faster onset and rate of recovery
B. Provide a state of conscious sedation
C. Are commonly used for induction of anesthesia
D. *All of the above
E. None of the above
297.
Indicate the intravenous anesthetic, which is an ultra-short-acting barbiturate:
A. Fentanyl
B. *Thiopental
C. Midazolam
D. Ketamine
E. Propofol
298.
Indicate the intravenous anesthetic, which is a benzodiazepine derivative:
A. *Midazolam
B. Thiopental
C. Ketamin
D. Propofol
E. Fentanyl
299.
Which of the following agents is used to accelerate recovery from the sedative
actions of intravenous benzodiazepines?
A. Naloxone
B. *Flumazenil
C. Ketamine
D. Fomepizole
E. None of the above
300.
Neuroleptanalgesia has all of the following properties EXCEPT:
A. Droperidol and fentanyl are commonly used
B. It can be used with nitrous oxide to provide neuroleptanesthesia
C. *Hypertension is a common consequence
D. Confusion and mental depression can occur as adverse effects
E. None of the above
301.
Which of the following intravenous anesthetics has antiemetic actions?
A. Thiopental
B. *Propofol
C. Ketamine
D. Fentanyl
E. All of the above
302.
Indicate the intravenous anesthetic, which causes minimal cardiovascular and
respiratory depressant effects:
A. Propofol
B. Thiopental
C. *Etomidate
D. Midazolam
E. Fentanyl
303.
Indicate the intravenous anesthetic, which produces dissociative anesthesia:
A. Midazolam
B.
C.
D.
E.
*Ketamine
Fentanyl
Thiopental
None of the above
304.
A 10-year-old girl had an attack of palpitation with nausea, dizziness, generalized
fatigue. On ECG: tachycardia with heartbeat rate of 218/min. Ventricle complexes are
deformed and widened. P wave is absent. What medication is to be prescribed to provide
first aid?
A. * Lydocain
B. Quinidine
C. Strophantin
D. Isoptin
305.
A 3-year-old child had an attack of palpitation with nausea, dizziness, generalized
fatigue. On ECG: tachycardia with heartbeat rate of 224/min. Ventricle complexes are
deformed and widened. P wave is absent. What medication is to be prescribed to provide
first aid?
A. * Novocainamides
B. Lydocain
C. Seduxen
D. Strophantin
E. Panangin
306.
A 44-year-old patient has ischemic heart disease, angina on exertion, II functional
class. What is the drug of choice in treatment of acute attack?
A. Platelet inhibiting agents
B. * Nitroglycerin sublingually or in spray form
C. Spasmolitics IV
D. Digitalis IV
E. Sedative agents orally
307.
A 45-year-old men with diagnosis: Paroxysmal arrhythmia. Arterial hypertension II.
Drug of choice for stopping attack:
A. * Metoprolol
B. Nitroglycerin
C. Potassium chloride
D. Digoxin
E. Lidocain
308.
A 45-year-old women with diagnosis: Paroxysmal arrhythmia. Arterial hypertension
II. Drug of choice for stopping attack:
A. * Anaprilin
B. Nitroglycerin
C. Potassium chloride
D. Digoxin
E. Lidocain
309.
A 48-year-old patient was admitted to resusitation department with diagnosi: aqute
myocardial infarction . Which one of the following drugs is uneffective in this case?
A. * Felodipine
B. Nitroglycerin
C. Metoprolol
D. Phentanyl
E. Morfin
310.
A 49-year-old patient has ischemic heart disease, angina on exertion, II functional
class. What is the drug of choice in treatment of acute attack?
A. No-spa IV
B. Aspirin
C. * Nitroglycerin sublingually
D. Digitalis IV
E. Seduxenum orally
311.
A 5-year-old child had an attack of palpitation with nausea, dizziness, generalized
fatigue. On ECG: tachycardia with heartbeat rate of 220/min. Ventricle complexes are
deformed and widened. P wave is absent. What medication is to be prescribed to provide
first aid?
A. * Lydocain
B. Seduxen
C. Novocainamides
D. Strophantin
E. Isoptin
312.
A 55-year-old men has stenocardia on exertion II. Taking of nitroglycerin potentiate
a sever headache. Which of the following drugs is the most useful in this case?
A. Aspirin
B. * Molsidomin
C. Captopril
D. Propranolol
E. Verapamil
313.
A 58-year-old women, who suffers from IHD, stenocardia on exertion II, heart
insufficiency I, has an attack of ventricular tachycardia, Ps 180/min, AP – 80/50 mm Hg.
Choose a correct treatment:
A. Digoxin, mezaton
B. * Lidocain, potassium chloride
C. Verapamil, potassium chloride
D. Digoxin
E. ATP, cordaron
314.
A 6-year-old child had an attack of palpitation with nausea, dizziness, generalized
fatigue. On ECG: tachycardia with heartbeat rate of 216/min. Ventricle complexes are
deformed and widened. P wave is absent. What medication is to be prescribed to provide
first aid?
A. Verapamil
B. Seduxen
C. Novocainamides
D. Strophantin
E. * Lydocain
315.
A 60-year-old man with unstable angina pectoris fails to respond to heparin,
nitroglycerin, beta adrenegic blockers and calcium channel antagonisst. What is the best
treatment for this patient?
A. Aspirin, orally
B. * Coronary artery bypass grafting
C. Antihypertensive therapy
D. Intravenous strpetokinase
E. Excercise testing
316.
A 60-year-old women has stenocardia on exertion II functional class. Taking of
nitroglycerin potentiate a sever headache. Which of the following drugs is the most useful in
this case?
A. Amiodaron
B. * Molsidomin
C. Nifedipine
D. Propranolol
E. Verapamil
317.
A 61-year-old patient complains of intensive and prolonged retrosternal pains,
decreased exercise tolerance for 4 days. Which of the following groops is the most useful?
A. ACE inhibitors
B.
C.
D.
E.
dyslipidemic drugs
beta-blockers
* nitrates
digitalis
318.
A 65-year-old man with unstable angina pectoris fails to respond to heparin,
nitroglycerin, beta adrenegic blockers and calcium channel antagonisst. What is the best
treatment for this patient?
A. Aspirin, orally
B. * Coronary artery bypass grafting
C. Digitalis
D. Intravenous strpetokinase
E. Excercise testing
319.
A 65-year-old men complains of intensive and prolonged retrosternal pains,
decreased exercise tolerance for 5 days. Which of the following groops is the most useful?
A. antiarrhythmic drugs
B. dyslipidemic drugs
C. diuretics
D. ACE inhibitors
E. * nitrates
320.
A 67 year old patient with a history of heart disease presents to the ER with chronic
atrial fibrillation, an average ventricular rate of 120-140 beats/min, and an ejection fraction
of 35%. The best drug for controlling his ventricular rate would be:
A. propranolol
B. quinidine
C. nifedipine
D. diltiazem
E. * digoxin
321.
A drug that can selectively supress automaticity in Purkinje fibers compared to the
sinus node:
A. verapamil
B. atenolol
C. diltiazem
D. propranolol
E. * lidocaine
322.
A drug that is NOT known to increase the effect or toxicity of digoxin when given
concominantly:
A. furosemide
B. verapamil
C. amiodarone
D. * propranolol
E. quinidine
323.
A drug used in the treatment of systolic congestive heart failure that shifts the Frank
Starling curve upwards and to the left, and produces its effects by inhibiting the cardiac
Na/K pump:
A. furosemide
B. nifedipine
C. captopril
D. clonidine
E. * digoxin
324.
A drug, or class of drugs, that has been documented to reduce mortality following a
myocardial infarction:
A. Class Ia antiarrhythmics
B. Class Ic antiarrhythmics
C. * Class II antiarrhythmics
D. Class III antiarrhythmics
E. digoxin
325.
A man 36 years old has arterial hypertension. Which group of drugs may be used
for her treatment?
A. Betaadrenomimetics
B. * Beta-blockers
C. Glucocorticoids
D. Preparations of calcium
E. M-cholinolitics
326.
A man 36, years old, has arterial hypertension. Which group of drugs may be used
for her treatment?
A. Betaadrenomimetics
B. Preparations of calcium
C. Glucocorticoids
D. * Calcium channel inhibitors
E. M-cholinolitics
327.
A patient being treated for heart failure with digoxin and furosemide is discharged
from the hospital after being initiated on quinidine therapy to reduce the incidence of atrial
tachyarrhythmias. The next day the patient reappears in your office complaining of fatigue,
visual difficulties and nausea. The patient's ECG reveals a prolong PR interval, with
occasional runs of bigeminy and 2nd degree (Mobitz type I) AV conduction block. The most
likely cause of these symptoms is:
A. furosemide induced hyperkalemia
B. an interaction between quinidine and furosemide
C. worsening heart failure
D. * an interaction between quinidine and digoxin
E. quinidine toxicity
328.
A patient has arterial hypertension with bradyarrhythmia. Which of the following
drugs is necessary to administer?
A. * Nifedipine
B. Klonidine
C. Propranolol
D. Diltiazem
E. Methyldopa
329.
A patient has bronchial asthma for a long time. Angina pectoris attacks have
appeared recently Which of the following drugs is contraindicated?
A. * Propranolol
B. Molsidomin
C. Nifedipine
D. Carvedilol
E. Aspirin
330.
A patient has chronic obstructive pulmonary desease for a long time. Angina pectoris
attacks have appeared recently. Which of the following drugs is contraindicated?
A. Nifedipine
B. Isosorbide dinitrate
C. * Propranolol
D. Lovastatin
E. Aspirin
331.
A patient of 42 year has arterial hypertension with bradyarrhythmia. Which of the
following drugs is necessary to administer?
A. Klonidine
B. * Nifedipine
C. Bisoprolol
D. Diltiazem
E. Methyldopa
332.
A patient of 53 year has arterial hypertension with tachycardia. Which of the
following drugs is necessary to administer?
A. Klonidine
B. Nifedipine
C. *Bisoprolol
D. Diltiazem
E. Methyldopa
333.
A patient who is experiencing progressive dyspnea of effort comes to your office for
a physical examination. The pulse is regular at 100 beats per minute, with a blood pressure
of 126/84 mm Hg. Auscultation of the lungs reveals inspiratory crackles at both bases. There
is no peripheral edema, and the cardiac apical impulse is not displaced. An echocardiogram
reveals a left ventricular chamber that is not dilated, with an estimated ejection fraction of
55%. You make the diagnosis of congestive heart failure. Based on your observations,
which of the following drugs would best address the problem in this patient?
A. dopamine
B. * verapamil
C. digoxin
D. hydralazine
E. furosemide
334.
A primary mechanism by which Class I antiarrhythmic drugs produce their
therapeutic effect in patients with ventricular arrhythmias:
A. Block L-type Ca channels
B. Block beta-adrenergic receptors
C. * Increase the Effective Refractory Period
D. Increase vagal tone to the AV node
E. Increase the dispersion of refractoriness
335.
A side effect of furosemide that can increase the automaticity of ectopic cardiac
pacemakers:
A. hypercalcemia
B. hyperkalemia
C. * hypokalemia
D. hyponatremia
E. hypothyroidism
336.
A woman 51 years old has arterial hypertension. Which group of drugs may be used
for her treatment?
A. Betaadrenomimetics
B. * Angiotensin-converting enzyme inhibitors
C. Glucocorticoids
D. Preparations of calcium
E. M-cholinolitics
337.
A woman 56 years old with hypertension edema develops on lower extremities,
moist wheezes in the lower parts of lungs. What must be administered in the complex
therapy of the patient?
A. Betaadrenomimetics
B. * Diuretics
C. Glucocorticoids
D. Preparations of calcium
E. M-cholinolitics
338.
Acebutolol is
A. * Beta-adrenoreceptor antagonist
B. Calcium channel blocker
C. Beta-adrenoreceptor agonist
D. Angiotensin II receptor antagonists
339.
340.
341.
342.
343.
344.
345.
346.
347.
E. ACE inhibitor
Adverse effects of organic nitrates are:
A. nausea and constipation
B. bradycardia, bronchospasm, heart failure
C. peripheral edema, headache, heart failure
D. * throbbing headache, dizziness, tachycardia
E. lightheadedness, weakness, peripheral edema
Adverse effects of nitrates are:
A. * hypotension, dizziness, tachycardia
B. bradycardia, bronchospasm, heart failure
C. peripheral edema, headache, heart failure
D. nausea and constipation
E. lightheadedness, weakness, peripheral edema
Adverse effects of beta-adrenergic blocking agents are:
A. * bradycardia, bronchospasm, heart failure
B. hypotension, dizziness, lightheadedness
C. headache, heart failure, pulmonary edema
D. nausea and constipation
E. hypotension, dizziness, tachycardia
Adverse effects of verapamil are:
A. * dizziness, lightheadedness, peripheral edema, constipation
B. bradycardia, bronchospasm, heart failure
C. hypotension, tachycardia, headache
D. palpitations and headache
E. hypotension, dizziness
Adverse effects of calcium channel blockers are:
A. hypotension, dizziness
B. bradycardia, bronchospasm, heart failure
C. hypotension, tachycardia, headache
D. palpitations and headache
E. * dizziness, lightheadedness, peripheral edema, constipation
Adverse effects of nifedipine are:
A. bradycardia, bronchospasm, heart failure
B. * dizziness, lightheadedness, peripheral edema, constipation
C. hypotension, tachycardia, headache
D. palpitations and headache
E. hypotension, dizziness
Agents that cause accumulation of cyclic AMP produce all of the following except:
A. increased cardiac contractility
B. * inhibition of Na+/K+-ATPase
C. increased uptake of Ca2+ by the sarcoplasmic reticulum in heart
D. increased entry of Ca2+ via voltage-operated L channels in heart
E. arteriolar vasodilation
Agents which may lower circulating plasma lipids include:
A. Clofibrate
B. Nicotinic acid
C. Cholestyramine
D. Dextrothyroxine
E. * All are correct
All of the following statements concerning nitroglycerin are correct EXCEPT:
A. It causes an elevation of intracellular cGMP
B. It undergoes significant first-pass metabolism in the liver.
C. It may cause significant reflex tachycardia.
D. * It significantly decrease AV conduction.
E. It can cause postural hypotension
All of the following statements concerning nitroglycerin are correct EXCEPT:
A. It causes an elevation of intracellular cGMP
B. It undergoes significant first-pass metabolism in the liver.
C. It may cause significant reflex tachycardia
D. * It can not cause throbbing headache
E. It can cause postural hypotension
349.
All of the following mechanisms of action correctly match a drug EXCEPT:
A. Quinidine: Blocks Na+ channels
B. Brethylium: Blocks K+ channels
C. Verapamil: Blocks Ca++ channels
D. Propranolol: Blocks beta adrenoceptors
E. * Procainamide: Blocks K+ channels
350.
All of the following mechanisms of action correctly match a drug EXCEPT:
A. Quinidine: Blocks Na+ channels
B. Brethylium: Blocks K+ channels
C. Verapamil: Blocks Ca++ channels
D. Propranolol: Blocks beta adrenoceptors
E. * Sotalol: Blocks K+ channels
351.
Amlodipine and verapamil both
A. * cause coronary vasodilation
B. depress atrioventricular nodal conduction
C. cause reflex tachycardia
D. stimulate calcium entry into cells
352.
An antiarrhythmic drug with the most "non-selective" mechanism of action:
A. lidocaine
B. quinidine
C. propranolol
D. sotalol
E. * amiodarone
353.
Antihypertensive drug LEAST likely to elevate serum lipids:
A. propranolol (Inderal)
B. metoprolol (Lopressor)
C. chlorothiazide (Diuril)
D. * diltiazem (Cardiazem)
E. all of the above
354.
Antihypertensive drugs belonging to the same class:
A. doxazosin (Cardura), prazosin (Minipress), metoprolol (Lopressor)
B. * nifedipine (Procardia, Adalat), verapamil (Isoptin, Calan), diltiazem (Cardiazem)
C. clonidine (Catapres), guanabenz (Wytensin),terazosin (Hytrin)
D. lisinopril (Prinvivil, Zestril), fosinopril (Monopril), guanadrel (Hylorel)
E. None of the above
355.
At the 46 years old patient with blink arrhythmia the edema of lungs began. What
drug is necessary to inject in the first turn?
A. * Furosemide
B. Triamterene
C. Verospirone
D. Amiloride
E. Euphylline
356.
Automaticity in Purkinje fibers can be most enhanced by:
A. vagal stimulation
B. hypercalcemia
C. hyponatremia
D. * catecholamines
348.
E. lidocaine
357.
Binds bile acids in the intestine, thus preventing their return to the liver via the
enterohepatic circulation:
A. Niacin
B. * Clofibrate
C. Cholestyramine
D. Probucol
E. Lovastatin
358.
Calcium channel blocker: vasodilation, less likely to have direct cardiac effects:
A. nifedipine (Procardia, Adalat)
B. diltiazem (Cardiazem)
C. * nifedipine & diltiazem
D. avoid vomitting
E. none of the above
359.
Calcium channel blockers with prolong action:
A. Nifedipine
B. Verapamil
C. Diltiazem
D. * Amlodipine
E. Nebivolol
360.
Call calcium channel blockers with prolong action:
A. * Felodipine
B. Verapamil
C. Diltiazem
D. Nifedipine
E. Nebivolol
361.
Name calcium channel blockers with prolong action:
A. * Isradipine
B. Nifedipine
C. Diltiazem
D. Verapamil
E. Nebivolol
362.
Cause a decrease in liver triacylglycerol synthesis by limiting available free fatty
acids needed as building blocks for this pathway:
A. Cholestyramine
B. * Niacin
C. Clofibrate
D. Probucol
E. Lovastatin
363.
Cause a decrease in liver triacylglycerol synthesis by limiting available free fatty
acids needed as building blocks for this pathway:
A. Simvastatin
B. Probucol
C. Clofibrate
D. * Niacin
E. Lovastatin
364.
Cause a decrease in plasma triacylglycerol levels by increasing the activity of
lipoprotein lipase:
A. Lovastatin
B. * Clofibrate
C. Cholestyramine
D. Niacin
E. Probucol
365.
Clonidine lowers blood pressure:
A.
B.
C.
D.
E.
by inhibiting alpha-1 adrenergic receptors
by inhibiting angiotensin converting enzyme
* by stimulating central alpha-2 adrenergic receptors
by inhibiting endothelin receptors
by inhibiting beta-adrenergic receptors
366.
Decreases automaticity of ectopic pacemakers
A. Quinidine
B. Lidocaine
C. * Quinidine & Lidocaine
D. Neither
E. Bisoprolol
367.
Diltiazem:
A. * decreases calcium entry through L type channels
B. increases total peripheral resistance
C. increases cardiac output
D. increases heart rate
E. increases gastrointestinal motility
368.
Drowsiness, paresthesias, muscle twitching, convulsions, changes in mental status
(eg, confusion), hypersensitivity reactions (eg, urticaria, edema, anaphylaxis) – adverse
effects of:
A. Diltiazem
B. * Lidocaine
C. Bisoprolol
D. Disopyramide
E. Verapamil
369.
Drug intervention for acute management of aortic dissection in which the objective is
to reduce
shear forces generated by blood flow:
A. hydralazine (Apresoline)
B. nitroprusside sodium (Nipride)
C. * nitroprusside sodium (Nipride) + propranolol (Inderal)
D. all of the above
E. none of the above
370.
During a treatment, an patient develops a drug-induced dry cough. The drug(s)
possibly responsible for this cough is (are):
A. propanolol
B. indomethacine
C. * captopril
D. furosemide
E. acetaminophen
371.
During treatment, a patient develops a drug-induced dry cough. The drugs possibly
responsible for this cough are:
A. codeine
B. diclofenac
C. verapamil
D. * perindopril
E. acetaminophen
372.
Each of the following agents is useful to treat diastolic heart failure EXCEPT:
A. lisinopril
B. * digoxin
C. verapamil
D. furosemide
E. carvedilol
373.
Equilibrium between plasma and tissue levels for amiodarone is reached:
A.
B.
C.
D.
E.
* in 1–3 wk
in 1 or 2 d
in approximately 1 wk
in just a few minutes
in 2 h
374.
Equilibrium between plasma and tissue levels for phenytoin is reached:
A. * in approximately 1 wk
B. in 1 or 2 d
C. in 1–3 wk
D. in just a few minutes
E. in 2 h
375.
Equilibrium between plasma and tissue levels for quinidine is reached:
A. * in 1 or 2 d
B. in approximately 1 wk
C. in 1–3 wk
D. in just a few minutes
E. in 2 h
376.
Equilibrium between plasma and tissue levels for IV lidocaine is reached:
A. * in just a few minutes
B. in 1 or 2 d
C. in approximately 1 wk
D. in 1–3 wk
E. in 2 h
377.
Extensively metabolized by circulating enzymes:
A. Amiodaron
B. Lidocaine
C. Both
D. * Neither
E. colestipol
378.
GI problems—nausea, vomiting, flatulence, constipation or diarrhea, abdominal
discomfort are the most common adverse effects of:
A. * dyslipidemic drugs
B. calcium channel blockers
C. beta-adrenergic blocking agents
D. nitrates
E. diuretic drugs
379.
GI upset, tinnitus and cinchonism are common side effects of:
A. lidocaine
B. digoxin
C. propranolol
D. * quinidine
E. amiodarone
380.
i.v. route of administration; few side effects; effective in treating hypertensive crisis:
A. nifedipine (Procardia, Adalat)
B. * nitroprusside sodium (Nipride)
C. nicardipine (Cardene)
D. all of the above
E. none of the above
381.
In a patient with a high risk for coronary artery disease (LDL cholesterol 200 mg/dL,
normal tryglycerides), the best lipid lowering drug would be:
A. nicotinic acid
B. gemfibrozil
C. atorvastatin
D. colestipol
E. * cholestyramine
382.
In the complex medical treatment of hypertensive disease a diuretic was prescribed
to the patient. In a few days the BP went down, but the signs of hypokaliaemia arose up.
What drug could cause such complication?
A. Triamterene
B. Clofeline
C. Enalapril
D. Spironolactone
E. * Furosemide
383.
In the patient with the considerable peripheral edema the by turns using of
dihlothiazid, ethacrynic acid and Furosemidee did not result in the considerable diuretic
effect. The analysis of blood indicated the considerable increasing of aldosteron level.
Prescribe drug for treatment.
A. Mannitol
B. * Spironolactone
C. Clopamid
D. Triamterene
E. Amiloride
384.
In the treatment of congestive heart failure, choose a drug that would reduce dyspnea
and pulmonary edema by decreasing the left ventricular preload:
A. hydralazine
B. propranolol
C. * isosorbide dinitrate
D. dobutamine
E. digoxin
385.
In the treatment of exertional angina:
A. * nitroglycerin reduces O2 demand by decreasing preload
B. nitroglycerin increases O2 supply by dilating atherosclerotic vessels
C. attacks frequently occur during sleep
D. nitroglycerin increases cardiac myocyte cAMP levels
E. beta blocking agents have no beneficial action
386.
Increases transmembrane action potential duration:
A. Lidocaine
B. * Quinidine
C. Atenolol
D. Digoxin
E. Esmolol
387.
Indicate the antiarrythmic drug which exerts membrane-stabilizing action.
A. Atenolol
B. * Quinidine sulfate
C. Nitroglycerin
D. Digoxin
E. Isadrinum
388.
Indicate the antiarrytmic drug from of adrenoblockers group
A. Nitroglycerin
B. Amiodaron
C. * Metoprolol
D. Verapamil
E. Asparcam
389.
Indicate the side effect of nitroglycerine
A. Itching of skin
B. Bradicardia
C. * Headache
D. Rise of arterial pressure
E. Intestine atony
Indicate the Verapamil possible side effect, as antiarrhythmic drug
A. Tachycardia
B. Bronchospasm
C. Stenocardia
D. Mouth dryness
E. * Decrease of artery pressure (hypotension)
391.
Inhibits 3-hydroxy-3-methylglutaryl CoA reductase, the rate-limiting step in
cholesterol synthesis:
A. * Pravastatin
B. Niacin
C. Cholestyramine
D. Clofibrate
E. Probucol
392.
Inhibits 3-hydroxy-3-methylglutaryl CoA reductase, the rate-limiting step in
cholesterol synthesis:
A. * Lovastatin
B. Niacin
C. Cholestyramine
D. Clofibrate
E. Probucol
393.
Inhibits 3-hydroxy-3-methylglutaryl CoA reductase, the rate-limiting step in
cholesterol synthesis:
A. * Atorvastatin
B. Niacin
C. Cholestyramine
D. Clofibrate
E. Probucol
394.
is a specific alpha-adrenergic antagonist:
A. * Prazosin
B. Timolol
C. Labetalol
D. Atenolol
E. Sotalol
395.
Isosorbide dinitrate:
A. Antiarrhythmic action is via beta-adrenergic blockade
B. Ventricular premature depolarizations
C. * Angina pectoris prophylaxis
D. Conversion of atrial fibrillation to sinus rhythm
E. Treatment of ventricular fibrillation
396.
Labetalol is
A. is a specific beta-adrenergic antagonist
B. * a selective beta-adrenergic antagonistas well as a nonspecific alpha-adrenergic
antagonist
C. is a nonspecific beta-adrenergic antagonist
D. is a selective beta1-adrenergic blocker
E. Neither
397.
Mechanism of action: prazosin (Minipress)
A. * alpha-1 receptor blocker
B. beta receptor blocker
C. phosphodiesterase inhibitor
D. calcium channel blocker
E. none of the above
398.
Mechanism of action:diltiazem (Cardiazem)
390.
A.
B.
C.
D.
E.
phosphodiesterase inhibitor
* blockade of calcium channels
alpha-1 receptor antagonists
beta-1 receptor antagonist
all of the above
399.
Nifedipine and verapamil both
A. * cause coronary vasodilation
B. depress atrioventricular nodal conduction
C. cause reflex tachycardia
D. stimulate calcium entry into cells
E. decreased vagal tone
400.
Nitrates relieve angina pain by reducing preload, which is:
A. * Oxygen demand of the heart
B. Pressure within the heart
C. Blood volume within the heart
D. Pressure against which the heart must pump
E. Contractility of the heart muscle
401.
Nitroderm is used:
A. Antiarrhythmic action is via ?-adrenergic blockade
B. Ventricular premature depolarizations
C. * Angina pectoris prophylaxis
D. Conversion of atrial fibrillation to sinus rhythm
E. Treatment of ventricular fibrillation
402.
Patients suffering from congestive heart failure will show signs and symptoms of
peripheral vasoconstriction, moist skin, pale complextion because of:
A. Na and water retention
B. decreased renin release
C. * increased sympathetic tone
D. decreased vagal tone
E. decreased aldosterone levels
403.
Primary indications for treatment of cardiac arrhythmias include all of the following
EXCEPT:
A. * Arrhythmias that reduce cardiac output
B. Arrhythmias that are prone to degenerate into more serious arrhythmias
C. Arrhythmias that cause vascular stasis
D. Arrhythmias that increase the risk of stroke
E. Monomorphic premature ventricular beats
404.
Sartan: the drugs whose international common name ends by the suffix - sartan are:
A. antineoplastic agents
B. antiemetic agents
C. antihypertensive drugs
D. * angiotensin II receptor antagonists
E. enkephalin receptor antagonists
405.
Selective blockade of beta1-adrenergic receptors:
A. Amiodaron
B. Lisinopril
C. Timolol
D. * Atenolol
E. Sotalol
406.
Side effects of this antihypertensive agent includes tachycardia, angina, reversible
lupus-like syndrome
A. propranolol (Inderal)
B. mecamylamine (Inversine)
C. * hydralazine (Apresoline)
D. diazoxide (Hyperstat)
E. none of the above
407.
Sodium nitroprusside:
A. * is used to lower blood pressure in a hypertensive emergency
B. increases smooth muscle cAMP levels
C. has a slow onset and long duration of action
D. is given orally
E. inhibits nitric oxide formation
408.
Tachycardia observed after minoxidil (Loniten) may be blocked by:
A. mecamylamine (Inversine)
B. propranolol (Inderal)
C. * Inversine & Inderal
D. avoid vomitting
E. none of the above
409.
The 56 years old woman with hypertensive disease appealed to the doctor.
Methyldopa was prescribed to her. Indicate a mechanism of action of this drug.
A. Violation of noradrenalin synthesis
B. * alfa2-adrenoreceptors agonist
C. Block of beta-adrenoreceptors
D. Depression of angiotensin converting enzyme activity
E. Increasing of acetylcholine synthesis
410.
The administration of digoxin to a patient with congestive heart failure results
eventually in a decrease in all of the following parameters EXCEPT
A. heart rate
B. end-diastolic pressure
C. arteriolar resistance
D. * AV node refractoriness
E. renal sodium and water retention
411.
The drug related to the group of calcium antagonists
A. Apressinum
B. Prazosinum
C. * Nifedipine
D. Reserpine
E. Papaverini hydrochloridum
412.
The drugs whose international common name ends in - sentan such as bosentan, are:
A. ACE (angiotensin converting enzyme) inhibitors
B. progestogens
C. * antagonists of endothelin receptors
D. agonists of adenosine receptors
E. directly acting cholinomimetic drugs
413.
The major drawback to antianginal use of atenolol is:
A. Diabetes-like hyperglycemia
B. Increased blood pressure
C. Urine retention
D. * Exacerbation of congestive heart failure
E. Blurred vision
414.
The major drawback to antianginal use of metoprolol is:
A. * Exacerbation of congestive heart failure
B. Increased blood presure
C. urine retention
D. blurred vision
E. diabetes-like hyperglycemia
415.
The major drawback to antianginal use of propranolol is:
A. * Exacerbation of congestive heart failure
B.
C.
D.
E.
416.
by:
A.
B.
C.
D.
E.
Increased blood pressure
urine retention
blurred vision
diabetes-like hyperglycemia
The mechanism by which digoxin causes a positive inotropic effect on heart tissue is
increasing ATP hydrolysis
stimulating the Na/K pump
* increasing intracellular Na and Ca
sensitizing the heart to catecholamines
inhibiting the Na/H exchanger
417.
The most common cellular mechanism responsible for production of cardiac
arrhythmias in patients with ischemic heart disease:
A. * hypokalemia
B. depressed automaticity
C. sick sinus syndrome
D. reentrant excitation
E. hormonal imbalance
418.
The patient has cranial trauma. The brain edema threat developed in the
postoperative period. To prescribe drug for the removal of this complication.
A. Spironolactone
B. Papaverini hydrochloride
C. * Furosemide
D. Dihlothiazid
E. Diacarb
419.
The patient suffer from hypertensive disease with bradiarrhythmia. What drug is
necessary to prescribe?
A. Papaverinum
B. Platyphyllini hydrotartras
C. Ceftriaxone
D. * Methyldopa
E. Paracetamol
420.
To indicate an antihypertensive drug from the calcium channels blockers.
A. Anaprilinum
B. Captopril
C. Dibazolum
D. * Verapamilum
E. Reserpinum
421.
To indicate the antihypertensive drug used for the medical treatment of hypertensive
disease with tachycardia and extrasystoles.
A. Reserpinum
B. Methyldopa
C. * Anaprilinum
D. Clophelinum
E. Dibazolum
422.
To indicate the possible side effect of Anaprilinum?
A. * Bronchial spasm
B. Tachycardia
C. Ortostatic collapse
D. Rise of АP
E. Development of tolerance
423.
To the 55 years old man for the medical treatment of gout was prescribed etamid.
What is the mechanism of uric actions of the drug?
A. * Depression of reabsorbing of urinary acid
B.
C.
D.
E.
Increasing of secretion of urinary acid
Decreasing of production of urinary acid
Depression of activity of ksantinoxydase
Production of easy soluble salts
424.
To the patient with edema of cardiac origin the doctor must prescribe diuretics. What
drugs are contra-indicated in this situation?
A. Hypothiazid
B. Spironolactone
C. Furosemide
D. * Mannitol
E. Diacarb
425.
Treatment of digoxin overdose may include all the following EXCEPT
A. withdrawal of digoxin
B. intravenous injection of Fab fragments of digitalis antibodies
C. potassium supplements
D. antiarrhythmic drugs
E. * cholestyramine to break enterohepatic circulation
426.
Vasoconstriction, aldosterone secretion, and renin release suppression occur upon
activation of the renin-angiotensin-aldosterone system. How would captopril (Capoten)
affect these responses?
A. * blocks all three
B. blocks only vasoconstriction
C. blocks all except vasoconstriction
D. no effect
E. none of the above
427.
Ventricular premature depolarizations
A. Isosorbide dinitrate
B. * Disopyramide
C. propranolol
D. Nifedipine
E. Nitroglycerin
428.
Weakness or dizziness, especially with activity or exercise – adverse effects of:
A. * Propranolol
B. Phenytoin
C. Lidocaine
D. Disopyramide
E. Verapamil
429.
Weight gain & edema occur in patients with congestive heart failure in response to:
A. * increased aldosterone levels
B. increased diuresis
C. decreased renin levels
D. decreased total peripheral resistance
E. decreased venous capacitance
430.
Weight gain & edema occur in patients with congestive heart failure in response to:
A. * increased aldosterone levels
B. increased diuresis
C. decreased renin levels
D. decreased total peripheral resistance
E. decreased venous capacitance
431.
What diuretic has potassium-sparing action?
A. Furosemide
B. Mannitol
C. Dihlothiazid
D. * Triamterene
432.
433.
434.
435.
436.
437.
438.
439.
440.
E. Urea
What diuretic is belonging to osmotic diuretics?
A. Spironolactone
B. * Mannitol
C. Furosemide
D. Dihlothiazid
E. Triamterene
What diuretic is used for forced diuresis?
A. Dihlothiazid
B. Triamterene
C. Spironolactone
D. * Furosemide
E. Diacarb
What diuretic is used for the removal of hypertensive crisis?
A. Dichlothiazide
B. * Furosemide
C. Spironolactonum
D. Mannitum
E. Triamteren
When quinidine is administered to a patient with atrial fibrilation
A. the ventricular rate may increase dangerously
B. arterial hypotension usually results if the drug is administered intravenously
C. thrombi attached to fibrillating atria may embolize
D. * all are correct
E. none of the above
Which of the following adverse effects is associated with nitroglycerin?
A. Hypertension
B. * Throbbing headache
C. Bradycardia
D. Sexual dysfunction
E. Anemia
Which of the following adverse effects is associated with nitroglycerin?
A. Hypertension
B. * Increasing of intracranial pressure
C. Bradycardia
D. Sexual dysfunction
E. Anemia
Which of the following agents decrease automaticity of ectopic pacemakers
A. Quinidine
B. Lidocaine
C. * Quinidine & Lidocaine
D. Neither
E. Digoxin
Which of the following agents extensively metabolized by circulating enzymes:
A. Quinidine
B. Lidocaine
C. * Neither
D. Albuterol
E. Timolol
Which of the following agents increase transmembrane action potential duration:
A. Digoxin
B. Novocainamid
C. Atenolol
D. * Quinidine
E. Esmolol
Which of the following agents is (are) pure beta1-adrenergic agonists?
A. * Bisoprolol
B. Sotalol
C. Albuterol
D. Isoproterenol
E. Timolol
442.
Which of the following agents is (are) pure beta1-adrenergic agonists?
A. Albuterol
B. Propranolol
C. * Metoprolol
D. Isoproterenol
E. Timolol
443.
Which of the following agents is selective for beta1-adrenergic receptor?
A. Metaproterenol
B. Propranolol
C. * Nebivolol
D. Terbutalin
E. Esmolol
444.
Which of the following drugs is LEAST apt to enhance the toxicity of digoxin?
A. verapamil
B. furosemide
C. * triamterene
D. hydrochlorothiazide
E. quinidine
445.
Which of the following maneuvers is the most likely to improve renal perfusion in a
patient treated for acute cardiac failure?
A. administration of phenylephrine
B. administration of propranolol
C. injection of esmolol
D. * administration of dopamine
E. administration of epinephrine
446.
Which one of the following is the most common side effect of antihyperlipidemic
drug therapy ?
A. Elevated blood pressure
B. * Gastrointestinal disturbance
C. Neurological problems
D. Heart palpitations
E. Migraine headaches
447.
Which one of the following drugs decreases de novo cholesterol synthesis by
inhibiting the enzyme 3-hydroxy-3-methylglutaryl CoA reductase?
A. Clofibrate
B. Niacin
C. Cholestyramine
D. * Lovastatin
E. Gemfibrozil
448.
Which one of the following drugs decreases de novo cholesterol synthesis by
inhibiting the enzyme 3-hydroxy-3-methylglutaryl CoA reductase?
A. Niacotinic acid
B. Clofibrate
C. Cholestyramine
D. * Pravastatin
E. Gemfibrozil
449.
Which one of the following drugs is useful in treating tachycardia?
441.
A.
B.
C.
D.
E.
Salmeterol
Isoproterenol
Phentolamine
* Metoprolol
Prazosin
450.
Which one of the following drugs is useful in treating tachycardia?
A. * Bisoprolol
B. Isoproterenol
C. Phentolamine
D. Phenoxybenzamine
E. Prazosin
451.
Which one of the following hyperlipidemias is characterized by elevated plasma
levels of chylomicrons and has no drug therapy available to lower the plasma lipoprotein
levels?
A. * Type I
B. Type II
C. Type III
D. Type IV
E. Type V
452.
Which one of the following is a specific alfa-adrenergic antagonist:
A. Atenolol
B. Timolol
C. Labetalol
D. * Terazosin
E. Sotalol
453.
Which one of the following statements about antiarrhythmic drugs is CORRECT?
A. * They may act by converting unidirectional block to a bidirectional block.
B. They often cause an increase in cardiac output
C. As a group they have mild side effects
D. They all affect Na+ channels in the cell membrane
E. They are equally useful in atrial and ventricular arrhythmias
454.
Which one of the following statements is INCORRECT?
A. Lidocaine must be given parenterally
B. * Lidocaine is used mainly for atrial arrhythmias
C. Procainamide is associated with a reversible lupus phenomenon.
D. Quinidine is active orally
E. All antiarrhythmic drugs can suppress cardiac contractions
455.
Which one of the following statements is INCORRECT?
A. Quinidine prolongs repolarization and the effective refractory period.
B. Mexiletine shortens repolarization and decreases the effective refractory period.
C. * Propranolol increases Phase 4 depolarization.
D. Verapamil shortens the duration of the action potential.
E. Amiodarone prolongs repolarization.
456.
Which one of the following statements is INCORRECT?
A. * Atenolol increases Phase 4 depolarization
B. Mexiletine shortens repolarization and decreases the effective refractory period.
C. Quinidine prolongs repolarization and the effective refractory period
D. Verapamil shortens the duration of the action potential
E. Amiodarone prolongs repolarization
457.
Action of the parathyroid hormone is:
A. Increased calcium and phosphate absorption in intestine (by increased 1,25dihydroxyvitamin D3 production)
B. Decreased calcium excretion and increased phosphate excretion in kidneys
458.
459.
460.
461.
462.
463.
464.
465.
C. In bone, calcium and phosphate resorption increased by high doses. Low doses may
increase bone formation.
D. *All of the above
E. None of the above
The following statements about the parathyroid hormone are true, EXCEPT:
A. The parathyroid hormone (PTH) is a single-chain peptide hormone composed of 84
amino acids
B. The parathyroid hormone increases calcium and phosphate absorption in intestine
(by increased 1,25C. dihydroxyvitamin D3 production)
D. The parathyroid hormone increases serum calcium and decreases serum phosphate
E. *The parathyroid hormone increases calcium excretion and decreases phosphate
excretion in kidneys
F. None of the above
Which of the following statements about calcitonin is true:
A. Calcitonin secreted by parafollicular cells of the mammalian thyroid is a single-chain
peptide hormone with 32 aminoacids
B. Effects of calcitonin are to lower serum calcium and phosphate by acting on bones
and kidneys.
C. Calcitonin inhibits osteoclastic bone resorption.
D. *All of the above
E. None of the above
Mechanism of action of calcitonin is:
A. Inhibits hydroxyapatite crystal formation, aggregation, and dissolution
B. *Raises intracellular cAMP in osteoclasts
C. Activates bone resorption
D. Inhibits macrophages
E. None of the above
Indications for calcitonin administration are the following, EXCEPT:
A. Hypercalcemia
B. Paget's disease
C. *Hypophosphatemia
D. Osteoporosis
E. None of the above
Side effect of calcitonin is:
A. Hypercalcemia
B. Metastatic calcifications
C. *Tetany
D. GI toxicity
E. All of the above
Side effect of calcitonin is:
A. Pruritus
B. *Hypotension
C. Fractures
D. Hypocalcemia
E. None of the above
Glucocorticoid hormones alter bone mineral homeostasis:
A. By antagonizing vitamin D-stimulated intestinal calcium transport
B. By stimulating renal calcium excretion
C. By increasing parathyroid hormone stimulated bone resorption
D. *By all of the above
E. By none of the above
Action of vitamin D3 is:
A. Increased calcium and phosphate absorption by 1,25-dihydroxyvitamin D3
B. Calcium and phosphate excretion may be decreased by 25-hydroxyvitamin D3 and
1,25-dihydroxyvitamin D3
C. Increased calcium and phosphate resorption by 1,25-dihydroxyvitamin D3; bone
formation may be increased by 25,24-dihydroxyvitamin D3
D. *All of the above
E. None of the above
466.
Route of administration of vitamin D3 is:
A. Subcutaneous
B. *Oral
C. Intravenous
D. Intranasal
E. Intraperitoneal
467.
Side effect of vitamin D3 is:
A. Defective bone mineralization
B. Metastatic calcifications
C. Hepatic toxicity
D. *Nephrolithiasis
E. None of the above
468.
Indication of vitamin D3 is:
A. Hypercalcemia
B. Paget's disease
C. *Hypophosphatemia
D. Osteomalacia
E. None of the above
469.
Route of administration of 25-hydroxyvitamin D3 (calcifediol) is:
A. *Oral
B. Subcutaneous
C. Intravenous
D. Intranasal
E. All of the above
470.
Side effect of 25-hydroxyvitamin D3 (calcifediol) is:
A. Hypercalcemia
B. Pruritus
C. GI toxicity
D. *All of the above
E. None of the above
471.
Indications for 1,25-dihydroxyvitamin D3 (calcitriol) administration are the
following, EXCEPT:
A. Hypocalcemia in chronic renal failure
B. Vitamin D-dependent rickets
C. Malabsorption of vitamin D from intestine
D. *Elevated skeletal turnover
E. None of the above
472.
Indication for 1,25-dihydroxyvitamin D3 (calcitriol) administration is:
A. Vitamin D resistance
B. Elevated skeletal turnover
C. Hypercalcemia of malignancy
D. *Hypophosphatemia
E. None of the above
473.
The following statement refers to 1,25-dihydroxyvitamin D3 (calcitriol):
A. When rapidity of action is required, 1,25-dihydroxyvitamin D3 (calcitriol), 0.25-1 μg
daily, is the vitamin D metabolite of choice, since it is capable of raising serum
calcium within 24-48 hours
474.
475.
476.
477.
478.
479.
480.
481.
482.
B. Calcitriol also raises serum phosphate, though this action is usually not observed
early in treatment
C. Undergoes enterohepatic circulation
D. *All of the above
E. None of the above
Side effect of dihydrotachysterol is:
A. Hepatic toxicity
B. General malaise
C. Lymphocytopenia
D. *Hypertension
E. All of the above
Route of administration of dihydrotachysterol is:
A. Intravenous
B. Subcutaneous
C. *Oral
D. Intranasal
E. All of the above
Indication for cholecalciferol administration is:
A. Hypercalcemia
B. Parathyroid hormone deficiency
C. Primary hyperparathyroidism
D. *Malabsorption of vitamin D from intestine
E. All of the above
The unwanted effect of cholecalciferol is:
A. Defective bone mineralization
B. Lymphocytopenia
C. *CNS toxicity
D. Metastatic calcifications
E. All of the above
The unwanted effect of dihydrotachysterol is:
A. Tetany
B. Anorexia
C. *CNS toxicity
D. Lymphocytopenia
E. All of the above
Indication for dihydrotachysterol administration is:
A. Parathyroid hormone resistance
B. Paget's disease
C. Increased osteolysis
D. *Hypophosphatemia
E. All of the above
Conditions associated with hypophosphatemia include:
A. Primary hyperparathyroidism
B. Vitamin D deficiency
C. Idiopathic hypercalciuria
D. *All of the above.
E. None of the above
Recommended phosphorus daily allowance is:
A. *900-1200 mg
B. 600-900 g
C. 25 g
D. 1.5-4 mg
E. 2,5 mg
Interactions with other drugs of phosphorus is:
A.
B.
C.
D.
E.
483.
A.
B.
C.
D.
E.
484.
A.
B.
C.
D.
E.
485.
A.
B.
C.
D.
E.
486.
A.
B.
C.
D.
E.
487.
A.
B.
C.
D.
E.
488.
A.
B.
C.
D.
E.
489.
A.
B.
C.
D.
E.
490.
A.
B.
C.
D.
Amiloride: decrease renal excretion
Glucocorticoids: decrease absorption
Loop diuretics: increase renal excretion
*Calcitonin: increases renal excretion
All are true
Indication for pamidronate administration is:
Failure of vitamin D formation in skin
Hypoparathyroidism
Elevated skeletal turnover
*Hypercalcemia
All of the above
Route of administration of pamidronate is:
Oral
Subcutaneous
Intranasal
*Intravenous
All of the above
Route of administration of alendronate is:
Intravenous
Subcutaneous
*Oral
Intranasal
All of the above
Indications of alendronate are the following, EXCEPT:
*Hypoparathyroidism
Glucocorticoid-induced osteoporosis
Paget's disease
Syndromes of ectopic calcification
None of the above
Indications for etidronate administration are the following, EXEPT:
Paget's disease
Osteoporosis
*Hypophosphatemia
Hypercalcemia
None of the above
Which of the following statements refers to etidronate:
Reduces osteoclast activity without significantly affecting osteoblasts; useful in
treatment of Paget's disease
Serum phosphorus concentrations should be monitored at least daily in case of oral
administration
2nd generation biphosphonate (amino-biphosphonate)
*Bioavailability increases with the administered dose
All of the above
The major causes of hypocalcemia in the adult are:
Hypoparathyroidism
Vitamin D deficiency
Renal failure and malabsorption
*All of the above
None of the above
The major causes of hypercalcemia in the adult are the following, EXCEPT :
Hyperparathyroidism
Cancer with or without bone metastases
*Renal failure and malabsorption
Hypervitaminosis D
491.
492.
493.
494.
495.
496.
497.
498.
E. All of the above
Which of the following statements refers to calcium:
A. Recommended Ca daily allowance for males: 1. 1-10 years: 800 mg 2. 11-18 years:
1200 mg 3. 19-50 years: 1000 mg 4. > 51 years: 1000 mg
B. Ca chloride is very irritating and can cause necrosis if extravasated
C. In achlorhydric patients calcium carbonate should be given with meals to increase
absorption or patient switched to calcium citrate, which is somewhat better absorbed
D. *All of the above
E. None of the above
Indication for calcium administration is:
A. Failure of formation of vitamin D in skin
B. Malabsorption of vitamin D from intestine
C. Hypercalcemia of malignancy
D. *Vitamin D deficiency
E. None of the above
Which of the calcium preparations is the most preferable for IV injection
A. Calcium gluceptate (0.9 meq calcium/mL)
B. *Calcium gluconate (0.45 meq calcium/mL)
C. Calcium chloride (0.68-1.36 meq calcium/mL)
D. All of the above
E. None of the above
Which of the oral calcium preparations is often the preparation of choice:
A. *Calcium carbonate (40% calcium)
B. Calcium lactate (13% calcium)
C. Calcium phosphate (25% calcium)
D. Calcium citrate (17% calcium)
E. None of the above
Interactions with other drugs of calcium is:
A. Ethanol: decreases absorption
B. Loop diuretics: increase renal excretion
C. Glucocorticoids: stimulate renal excretion
D. *All of the above
E. None of the above
Correct statements about magnesium include all of the following, EXCEPT:
A. Magnesium is mainly an intracellular cation, and is the fourth most abundant cation
in the body
B. The recommended dietary amounts of magnesium have been set at 6 mg/kg day
(350-400 mg)
C. The most common specific causes encountered in clinical practice are: diet,
alcoholism (drinking), diarrhea and malabsorption, diabetes mellitus, diuretics, and
drugs such as aminoglycosides and amphotericin
D. *It is a physiological calcium agonist
E. All of the above
Recommended magnesium daily allowance is:
A. *350-400 mg
B. 6-9 g
C. 25 g
D. 1.5-4 mg
E. 150-200 mg
The major causes of hypomagnesaemia are:
A. Insufficient dietary intake, e.g. malnutrition
B. Abnormal gastrointestinal loss, e.g. severe diarrhea or chronic alcoholism
499.
500.
501.
502.
503.
504.
505.
506.
C. Abnormal renal loss, e.g. diabetes mellitus or during therapy with some kind of
drugs such as amphotericin B, gentamicin, cisplatin, cardiac glycosides, distal and
loop diuretics
D. *All of the above
E. None of the above
Which of the magnesium preparation is the most preferable for I.V. injection
A. Magnesium sulfate
B. *Magnesium chloride
C. Magnesium glutamate
D. All of the above
E. None of the above
Which of the oral magnesium preparations is often the preparation of choice:
A. Magnesium lactate
B. Magnesium oxide
C. *MagneB6 (Mg pidolate / Mg lactate + pyridoxine hydrochloride)
D. All of the above.
E. None of the above
Recommended fluoride daily allowance is:
A. *1.5-4 mg
B. 600-900 g
C. 25 g
D. 350-400 mg
E. 50-75 mg
Which of the following statements refers to gallium nitrate:
A. It is approved by the FDA for the management of hypercalcemia of malignancy
B. This drug acts by inhibiting bone resorption
C. Because of potential nephrotoxicity, patients should be well-hydrated and have good
renal output before starting the infusion
D. *All of the above
E. None of the above
Which of the following statements refers to plicamycin (formerly mithramycin):
A. Duration of action is usually several days
B. Mechanism of cytotoxic action appears to involve its binding to DNA, possibly
through an antibiotic-Mg2+ complex.
C. The drug causes plasma calcium levels to decrease, apparently through an action on
osteoclasts that is independent of its action on tumor cells and useful in
hypercalcemia.
D. *All of the above.
E. None of the above
Unwanted effects of plicamycin (formerly mithramycin) are the following, EXEPT:
A. Thrombocytopenia
B. GI toxicity
C. Bleeding disorders
D. *Fractures
E. Diarrhea
Unwanted effect of plicamycin (formerly mithramycin) is:
A. Diarrhea
B. *Myelosuppression
C. Nephrolithiasis
D. Metastatic calcifications
E. All of the above
Indication for plicamycin (formerly mithramycin) administration is:
A. Testicular cancers refractory to standard treatment
B. Paget’s disease
507.
C. Hypercalcemia of malignancy
D. *All of the above
E. None of the above
Route of administration of plicamycin is:
A. *Intravenous
B. Subcutaneous
C. Intranasal
D. Oral
E. All of the above
Situational questions.
1. The combination of hydralazine and nitrates has been shown to improve survival in patients of
heart failure. All of the following statements about this combination are true except:
A. The combination serves to decrease both afterload and preload.
B. *Prazosin is as effective as the combination in treatment of congestive heart failure.
C. The concept of afterload reduction is principally derived from patients of significant mitral
regurgitation.
D. The VA cooperative study was a landmark trial demonstrating the beneficial effect of
hydralazine and nitrate combination in patients of heart failure.
E. None of the above
2. A 45-year-old woman has had recurrent episodes of atrial fibrillation. She is receiving
phenytoin and quinidine to control the atrial fibrillation. She is also taking a low dose of
diazepam for insomnia and estrogen replacement therapy.You learn today that she has been
receiving ciprofloxacin for a urinary track infection.The reason for her appointment today is that
she has been having ringing in the ears, headache, nausea, and blurred vision. She tells you that
she is also having trouble hearing the television. You suspect drug toxicity.The most likely
agent is:
A. Ciprofloxacin
B. Estrogen
C. Phenytoin
D. Diazepam
E. *Quinidine
3. You are asked to treat a 55-year-old patient for continuing ventricular arrhythmias.The patient is
receiving timolol drops for glaucoma, daily insulin injections for diabetes mellitus, and an ACE
inhibitor for hypertension.You decide to use phenytoin instead of procainamide because of what
pharmacological effect of procainamide?
A. The local anesthetic effect of procainamide would potentiate diabetes.
B. *The anticholinergic effect of procainamide would aggravate glaucoma.
C. The hypertensive effects of procainamide would aggravate the hypertension.
D. The local anesthetic effect of procainamide would aggravate the hypertension.
E. The cholinergic effects of procainamide would aggravate the diabetes.
4. Exercise-induced ventricular tachycardia in persons without overt cardiac disease is an example
of delayed after-depolarizations and is characterized by an increase in intracellular ionized
calcium.This type of arrhythmia is known to often respond well to which of the following
combinations?
A. Beta-Blocker and ACE inhibitor
B. *Calcium channel antagonist and ACE inhibitor
C. Beta-blocker and ACE inhibitor
D. Beta-blocker and calcium channel antagonist
E. Alpa-blocker and calcium channel antagonist
5. Metoprolol would produce which beneficial effect in a patient with secondary angina?
A. A decrease in preload
B. An increase in collateral blood flow
C. An increase in afterload
D. *An increase in diastolic filling time
E. An increase in blood flow through a concentric stenosis
6. Although most antiarrhythmic drugs (and indeed most drugs) are chemically synthesized, some
compounds that occur endogenously in humans are useful. Indicate which of the following
agents occurs endogenously and is a useful antiarrhythmic agent.
A. Phenytoin
B. Digoxin
C. *Adenosine
D. Quinine
E. Lidocaine
7. A patient comes to your office with effort-induced angina and resting tachycardia.You choose
the following drug to treat the patient because it slows heart rate by blocking L-type calcium
channels in the SA node:
A. *Verapamil
B. Propranolol
C. Nitroglycerin
D. Isosorbide dinitrate
E. Metoprolol
8. Which of the following hemodynamic effects of nitroglycerin are primarily responsible for the
beneficial results observed in patients with secondary angina?
A. Reduction in the force of myocardial contraction
B. Reduction in systemic vascular resistance (afterload)
C. Increased heart rate
D. *Reduction in venous capacitance (preload)
E. Increased blood flow to the subepicardium
9. A woman is prescribed a combination of drugs consisting of a nitroglycerin patch and a betablocker, such as propranolol, to treat her attacks of secondary angina.Which effect of
propranolol would counteract an adverse effect of nitroglycerin?
A. A decrease in preload
B. A decrease in afterload
C. *A decrease in heart rate
D. An increase in myocardial contractile force
E. A reduction in coronary vasospasm
10. A patient who has been taking propranolol for a long period for secondary angina comes to your
office complaining of increased frequency of chest pains on exertion.You decide to stop the
propranolol and give him diltiazem because you suspect he has a mixture of secondary and
primary angina. Why would diltiazem be more likely to relieve the angina if your new diagnosis
is accurate?
A. Diltiazem produces a decrease in heart rate.
B. *Diltiazem dilates coronary blood vessels in spasm.
C. Diltiazem produces AV blockade.
D. Diltiazem reduces myocardial contractility.
E. Diltiazem reduces afterload.
11. An accurate statement regarding the actions of both ACE inhibitors and AT1 receptor
antagonists is that
A. Both classes of drugs increase bradykinin.
B. Angiotensin II can act at the AT2 receptor with both classes of drugs.
C. *Both classes of drugs reduce total peripheral resistance.
D. Both classes of drugs decrease circulating angiotensin II levels.
E. Both classes of drugs are first-choice treatments for congestive heart failure.
12. Angiotensin II can:
A. *Increase the synthesis and release of aldosterone
B. Reduce the activity of the sympathetic nervous system
C. Be a potent positive inotropic at the heart
D. Relax vascular smooth muscle
E. Reduce the growth of cardiovascular cell types
13. The most potent vasoconstrictor known is
A. Bradykinin
B. Angiotensin II
C. Angiotensin IV
D. Natriuretic peptide
E. *Endothelin
14. The mechanism of action of captopril is
A. Angiotensin receptor antagonist
B. *ACE inhibitor
C. Aldosterone receptor antagonist
D. Bradykinin antagonist
E. None of the above
15. Which of the following antihypertensive agents would decrease renin release?
A. Prazosin
B. *Clonidine
C. Captopril
D. Nitroprusside
E. Diazoxide
16. Which of the following statements most accurately characterize the cellular action of the
calcium channel blockers?
A. Their interaction with membrane phospholipids results in a nonselective decrease of ion
transport.
B. They inhibit the Na+–Ca++ exchanger in cardiac and smooth muscle.
C. *They interact at three distinct sites at the L-type voltage-gated calcium channels.
D. Their interaction with the sodium pump results in an inhibition of calcium transport.
E. All of the above
17. Which of the following calcium channel blockers would be most likely to suppress atrial
tachyarrhythmias involving the A-V node?
A. Nifedipine
B. *Verapamil
C. Nicardipine
D. Amlodipine
E. Nimodipine
18. All of the following statements are applicable with regard to the systemic effects caused by
nifedipine EXCEPT:
A. It typically causes peripheral vasodilation.
B. It often elicits reflex tachycardia.
C. It causes coronary vasodilatation
D. It increases coronary blood flow.
E. *Its benefit in the management of angina is related to the reduction in preload that it induces.
19. All of the following statements regarding the pharmacokinetics of calcium channel blockers are
correct EXCEPT
A. They are characterized by significant amount (~ 90%) of protein binding.
B. They undergo significant first-pass metabolism.
C. *Their half-life is not altered by hepatic cirrhosis.
D. They can be administered orally.
E. All of the above
20. All of the following adverse effects are likely to occur with long-term use of calcium channel
blockers EXCEPT
A. *Skeletal muscle weakness
B. Flushing
C. Dizziness
D. Headache
E. None of the above
21. A 55-year-old patient has been referred to you. She complains about a skin rash and a cough. In
the course of history taking, she tells you that she takes high blood pressure medication but she
doesn’t remember the name.You suspect a drug toxicity. Which of the following
antihypertensive agents is the patient most likely taking?
A. *Captopril
B. Nifedipine
C. Prazosin
D. Propanolol
E. Clonidine
22. Which of the following compounds depends least upon the release of EDRF (nitric oxide) from
endothelial cells to cause vasodilation?
A. Bradykinin
B. Histamine
C. *Minoxidil
D. Hydralazine
E. Acetylcholine
23. Which of the following antihypertensive drugs is contraindicated in a hypertensive patient with
a pheochromocytoma?
A. Metyrosine
B. Labetalol
C. Prazosin
D. Phenoxybenzamine
E. *Guanethidine
24. When a patient is treated with a thiazide diuretic for hypertension, all of the following are likely
EXCEPT:
A. The fall of blood pressure that occurs in the first 2 weeks of therapy results from a decrease
of extracellular volume.
B. The sustained fall in blood pressure that occurs after several weeks of therapy is due to a
decrease of intravascular resistance.
C. After the blood pressure is reduced, hypokalemia remains a complication.
D. Hyperuricemia may occur.
E. *Hypoglycemia may occur.
25. Furosemide increases the excretion of all of the following EXCEPT:
A. Na+
B. K+
C. Ca++
D. Mg++
E. *Uric acid
26. Which of the following drugs is an appropriate initial antihypertensive therapy in an otherwise
healthy adult with mild hypertension?
A. Bumetanide
B. Triamterene
C. *Hydrochlorothiazide
D. Aldactone
E. Furosemid
27. When furosemide is administered to a patient with pulmonary edema, there is often
symptomatic relief within 5 minutes of starting treatment.This relief is primarily due to:
A. A rapid diuretic effect
B. *An increase in venous capacitance
C. A direct effect on myocardial contractility
D. Psychological effects
E. All of the above
28. A 60-year-old man with unstable angina pectoris fails to respond to heparin, nitroglycerin, beta
adrenegic blockers and calcium channel antagonisst. What is the best treatment for this patient?
A. Aspirin, orally
B. *Coronary artery bypass grafting
C. Antihypertensive therapy
D. Intravenous strpetokinase
E. Excercise testing
29. A 5-year-old child had an attack of palpitation with nausea, dizziness, generalized fatigue. On
ECG: tachycardia with heartbeat rate of 220/min. Ventricle complexes are deformed and
widened. P wave is absent. What medication is to be prescribed to provide first aid?
A. *Lydocain
B. Seduxen
C. Novocainamides
D. Strophantin
E. Isoptin
30. A 46-year-old patient has ischemic heart disease, angina on exertion, II functional class. What is
the drug of choice in treatment of acute attack?
A. *Nitroglycerin sublingually
B. Platelet inhibiting agents (aspirin)
C. Spasmolitics (No-spa) IV
D. Digitalis IV
E. Sedative agents (Seduxenum) orally
31. A 52-year-old patient complains of intensive and prolonged retrosternal pains, decreased
exercise tolerance for 5 days. Which of the following groops is the most useful?
A. *nitrates
B. dyslipidemic drugs
C. diuretics
D. ACE inhibitors
E. digitalis
32. A 50-year-old patient was admitted to resusitation department with aqute myocardial infarction
. Which one of the following drugs is uneffective in this case?
A. Proranolol
B. Nitroglycerin
C. *Nifedipine
D. Phentanyl
E. Morfin
33. A 54-year-old patient, who suffers from IHD, stable stenocardia II, heart insufficiency I, has an
attack of ventricular tachycardia, Ps 180/min, AP – 80/50 mm Hg. Choose a correct treatment:
A. *Lidocain, potassium chloride
B. Mezaton, strofantin
C. Verapamil, potassium chloride
D. Digoxin
E. ATP, cordaron
34. A 55-year-old men has stenocardia on exertion II. Taking of nitroglycerin potentiate a sever
headache. Which of the following drugs is the most useful in this case?
A. *Molsidomin
B. Amiodaron
C. Nifedipine
D. Propranolol
E. Verapamil
35. A 49-year-old patient has ischemic heart disease, angina on exertion, II functional class. What is
the drug of choice in treatment of acute attack?
A. No-spa IV
B. Aspirin
C. Digitalis IV
D. *Nitroglycerin sublingually
E. Seduxenum orally
36. A patient of 42 year has arterial hypertension with bradyarrhythmia. Which of the following
drugs is necessary to administer?
A. Clonidine
B. Bisoprolol
C. *Nifedipine
D. Diltiazem
E. Methyldopa
37. A 58-year-old women, who suffers from IHD, stenocardia on exertion II, heart insufficiency I,
has an attack of ventricular tachycardia, Ps 180/min, AP – 80/50 mm Hg. Choose a correct
treatment:
A. Digoxin, mezaton
B. *Lidocain, potassium chloride
C. Verapamil, potassium chloride
D. Digoxin
E. ATP, cordaron
38. A 55-year-old men has stenocardia on exertion II. Taking of nitroglycerin potentiate a sever
headache. Which of the following drugs is the most useful in this case?
A. Aspirin
B. *Molsidomin
C. Captopril
D. Propranolol
E. Verapamil
39. A 6 month old patient who is in the ICU following cardiac surgery for a congenital defect
develops atrial flutter with an atrial rate of 300 beats/min and second degreee (2:1) AV node
block. A 3rd year medical student inadvertantly administers quinidine, instead of
propranolol,and the patient's ventricular rate skyrockets to 300 beats/min. The patient faints.
This most likely occured because quinidine is:
A. a partial beta agonist
B. *antimuscarinic
C. a sodium channel blocker
D. a potassium channel blocker
E. able to increase vagal tone
40. A 45-year-old women with diagnosis: Paroxysmal arrhythmia. Arterial hypertension II. Drug
of choice for stopping attack:
A. *Anaprilin
B. Nitroglycerin
C. Potassium chloride
D. Digoxin
E. Lidocain
41. A patient has arterial hypertension with bradyarrhythmia. Which of the following drugs is
necessary to administer?
A. *Nifedipine
B. Klonidine.
C. Propranolol
D. Diltiazem
E. Methyldopa
42. A patient has bronchial asthma for a long time. Angina pectoris attacks have appeared recently
Which of the following drugs is contraindicated?
A. *Propranolol
B. Nitroglycerin
C. Nifedipine
D. Lovastatin
E. Aspirin
43. A 64-year-old patient, who suffers from IHD, stable stenocardia II functional class, heart
insufficiency I, has an attack of ventricular tachycardia, Ps 180/min, AP – 80/50 mm Hg.
Choose a correct treatment:
A. ATP, cordaron
B. Mezaton, strofantin
C. Verapamil, potassium chloride
D. Digoxin
E. *Lidocain, potassium chloride
44. A 45-year-old men with diagnosis: Paroxysmal arrhythmia. Arterial hypertension II. Drug of
choice for stopping attack:
A. *Metoprolol
B. Nitroglycerin
C. Potassium chloride
D. Digoxin
E. Lidocain
45. A patient of 42 year has arterial hypertension with bradyarrhythmia. Which of the following
drugs is necessary to administer?
A. Clonidine
B. *Nifedipine
C. Bisoprolol
D. Diltiazem
E. Methyldopa
46. A patient has bronchial asthma for a long time. Angina pectoris attacks have appeared recently.
Which of the following drugs is contraindicated?
A. *Propranolol
B. Molsidomin
C. Nifedipine
D. Carvedilol
E. Aspirin
47. A 65-year-old woman with unstable angina pectoris fails to respond to heparin, nitroglycerin,
beta adrenegic blockers and calcium channel antagonists. What is the best treatment for this
patient?
A. Intravenous strpetokinase
B. *Coronary artery bypass grafting
C. Antihypertensive therapy
D. Aspirin, orally
E. Excercise testing
48. A 49-year-old women with diagnosis: Paroxysmal arrhythmia. Arterial hypertension II, heart
insufficiency 0. Introduce drug of choice for stopping attack:
A. *Metoprolol
B. Nitroglycerin
C. Potassium chloride
D. Digoxin
E. Lidocain
49. A patient suffers from arterial hypertension with bradyarrhythmia. Which of the following
drugs is necessary to administer?
A. Atenolol
B. Furosemid.
C. *Nifedipine
D. Diltiazem
E. Methyldopa
50. A patient has chronic obstructive pulmonary desease for a long time. Angina pectoris attacks
have appeared recently. Which of the following drugs is contraindicated?
A. Nifedipine
B. Isosorbide dinitrate
C. Lovastatin
D. *Propranolol
E. Aspirin
51. A woman 56 years old with hypertension edema develops on lower extremities, moist wheezes
in the lower parts of lungs. What must be administered in the complex therapy of the patient?
A. Beta-adrenomimetics,
B. *Diuretics,
C. Glucocorticoids,
D. Preparations of calcium,
E. M-cholinolitics.
52. A woman 51 years old has arterial hypertension. Which group of drugs may be used for her
treatment?
A. Betaadrenomimetics,
B. *Angiotensin-converting enzyme inhibitors,
C. Glucocorticoids,
D. Preparations of calcium,
E. M-cholinolitics.
53. A man 36, years old, has arterial hypertension. Which group of drugs may be used for her
treatment?
A. Beta-adrenomimetics
B. Preparations of calcium,
C. Glucocorticoids,
D. *Calcium-channel inhibitors,
E. M-cholinolitics.
54. Patients suffering from congestive heart failure will show signs and symptoms of peripheral
vasoconstriction, moist skin, pale complextion because of:
A. Na and water retention
B. decreased renin release
C. *increased sympathetic tone
D. decreased vagal tone
E. decreased aldosterone levels
55. A 67 year old patient with a history of heart disease presents to the ER with chronic atrial
fibrillation, an average ventricular rate of 120-140 beats/min, and an ejection fraction of 35 %.
The best drug for controlling his ventricular rate would be:
A. propranolol
B. quinidine
C. nifedipine
D. diltiazem
E. *digoxin
56. A 76 year old patient being treated with digoxin, captopril and furosemide for systolic heart
failure and hypertension presents to the ER with complaints of dizziness and fatigue. The
patient faints immediately after an ECG recording indicates the presence of sinus bradycardia
and intermittent 3rd degree AV conduction block. The most likely explanation for this patient's
rhythm disturbance is:
A. captopril overdose
B. *increased vagal tone caused by digoxin
C. decreased vagal tone caused by furosemide
D. increased sympathetic tone caused by digoxin
E. hyperkalemia caused by furosemide
57. A 59 year old patient with a history of systolic heart failure arrives in the ER with complaints of
chest palpitations, nausea and GI upset. His chart indicates that he is currently taking digoxin,
captopril, metoprolol and furosemide. A 12 lead ECG is recorded and reveals the presence of
frequent runs of ventricular bigeminy. His echocardiogram indicates an ejection fraction of
40%. His blood tests indicate a serum potassium level of 4.0 mM, and an elevated digoxin level
of 2.2 ng/ml. The most appropriate drug for treating this patient's arrhythmia would be:
A. propranolol
B. quinidine
C. *lidocaine
D. verapamil
E. atropine
58. A patient being treated for heart failure with digoxin and furosemide is discharged from the
hospital after being initiated on quinidine therapy to reduce the incidence of atrial
tachyarrhythmias. The next day the patient reappears in your office complaining of fatigue,
visual difficulties and nausea. The patient's ECG reveals a prolong PR interval, with occasional
runs of bigeminy and 2nd degree (Mobitz type I) AV conduction block. The most likely cause
of these symptoms is:
A. furosemide induced hyperkalemia
B. an interaction between quinidine and furosemide
C. worsening heart failure
D. *an interaction between quinidine and digoxin
E. quinidine toxicity
59. A patient who is experiencing progressive dyspnea of effort comes to your office for a physical
examination. The pulse is regular at 100 beats per minute, with a blood pressure of 126/84 mm
Hg. Auscultation of the lungs reveals inspiratory crackles at both bases. There is no peripheral
edema, and the cardiac apical impulse is not displaced. An echocardiogram reveals a left
ventricular chamber that is not dilated, with an estimated ejection fraction of 55%. You make
the diagnosis of congestive heart failure. Based on your observations, which of the following
drugs would best address the problem in this patient?
A. dopamine
B. *verapamil
C. digoxin
D. hydralazine
E. furosemide
60. The 56 years old woman with hypertensive disease appealed to the doctor. Methyldopa was
prescribed to her. Indicate a mechanism of action of this drug.
A. Violation of noradrenalin synthesis
B. *alpha-adrenoreceptors stimulator
C. Block of beta-adrenoreceptors
D. Depression of angiotensin converting enzyme activity
E. Increasing of acetylcholine synthesis
61. In the complex medical treatment of hypertensive disease a diuretic was prescribed to the
patient. In a few days the BP went down, but the signs of hypokaliaemia arose up. What drug
could cause such complication?
A. Triamterene
B. Clofeline
C. Enalapril
D. Spironolactone
E. *Furosemide
62. The patient has cranial trauma. The brain edema threat developed in the postoperative period.
To prescribe drug for the removal of this complication.
A. Spironolactone
B. Papaverini hydrochloride
C. *Furosemide
D. Dihlothiazid
E. Diacarb
63. In the patient with the considerable peripheral edema the by turns using of dihlothiazid,
ethacrynic acid and furosemide did not result in the considerable diuretic effect. The analysis of
blood indicated the considerable increasing of aldosteron level. Prescribe drug for treatment.
A. Mannitol
B. *Spironolactone
C. Clopamid
D. Triamterene
E. Amiloride
64. To the 55 years old man for the medical treatment of gout was prescribed etamid. What is the
mechanism of uric actions of the drug?
A. *Depression of reabsorbing of urinary acid
B. Increasing of secretion of urinary acid
C. Decreasing of production of urinary acid
D. Depression of activity of xantinoxydase
E. Production of easy soluble salts
65. At the 46 years old patient with blink arrhythmia the edema of lungs began. What drug is
necessary to inject in the first turn?
A. *Furosemide
B. Triamterene
C. Verospirone
D. Amiloride
E. Euphylline
66. To the patient with edema of cardiac origin the doctor must prescribe diuretics. What drugs are
contraindicated in this situation?
A. Hypothiazide
B. Spironolactone
C. Furosemide
D. *Mannitol
E. Diacarb
67. In the patient with the considerable peripheral edema using of dihlothiazid did not result in the
considerable diuretic effect. The analysis of blood indicated the considerable increasing of
aldosteron level. Prescribe drug for treatment in this situation.
A. Mannitol
B. *Spironolactone
C. Ethacrynic acid
D. Furosemide
E. Diacarb
68. In the complex medical treatment of hypertensive disease a diuretic was prescribed to the
patient. In a few days the BP went down, but the signs of hypokaliaemia arose up. What drug
could cause such complication?
A. Triamterene
B. Clofeline
C. Enalapril
D. Spironolactone
E. *Furosemide
69. Patients suffering from congestive heart failure will show signs and symptoms of peripheral
vasoconstriction, moist skin, pale complextion because of:
A. Na and water retention
B. decreased renin release
C. *increased sympathetic tone
D. decreased vagal tone
E. decreased aldosterone levels
70. A 67 year old patient with a history of heart disease presents to the ER with chronic atrial
fibrillation, an average ventricular rate of 120-140 beats/min, and an ejection fraction of 35%.
The best drug for controlling his ventricular rate would be:
A. verapamil
B. quinidine
C. nifedipine
D. diltiazem
E. *digoxin
71. A 76 year old patient being treated with digoxin, captopril and furosemide for systolic heart
failure and hypertension presents to the ER with complaints of dizziness and fatigue. The
patient faints immediately after an ECG recording indicates the presence of sinus bradycardia
and intermittent 3rd degree AV conduction block. The most likely explanation for this patient's
rhythm disturbance is:
A. captopril overdose
B. *increased vagal tone caused by digoxin
C. decreased vagal tone caused by furosemide
D. increased sympathetic tone caused by digoxin
E. hyperkalemia caused by furosemide
72. A patient being treated for heart failure with digoxin and furosemide is discharged from the
hospital after being initiated on quinidine therapy to reduce the incidence of atrial
tachyarrhythmias. The next day the patient reappears in your office complaining of fatigue,
visual difficulties and nausea. The patient's ECG reveals a prolong PR interval, with occasional
runs of bigeminy and 2nd degree (Mobitz type I) AV conduction block. The most likely cause
of these symptoms is:
A. furosemide induced hyperkalemia
B. an interaction between quinidine and furosemide
C. worsening heart failure
D. *an interaction between quinidine and digoxin
E. quinidine toxicity
73. An elderly male patient has essential hypertension, congestive heart failure, and type I insulindependent diabetes. His congestive failure developed secondary to coronary vascular disease
associated with hyperlipidemia. What antihypertensive drug(s) may be most appropriate for this
patient?
A. Chlorothiazide (Diuril)
B. *captopril (Capoten)
C. propranolol (Inderal)
D. metoprolol (Lopressor)
E. none of the above
74. An elderly male patient has essential hypertension, congestive heart failure, and type I insulindependent diabetes. His congestive failure developed secondary to coronary vascular disease
associated with hyperlipidemia. What antihypertensive drug(s) may be most appropriate for this
patient?
A. chlorothiazide
B. *lisinopril
C. propranolol
D. metoprolol
E. all of the above
75. A patient presents with a blood pressure of 160/110 mm Hg. The patient has a history of
coronary vascular disease, resulting in angina, but has no evidence of congestive heart failure.
The patient also has asthma and has been treated mainly using terbutaline (Brethine), by aerosol
inhalation Propranolol (Inderal) was prescribed to manage essential hypertension. Was this
action appropriate?
A. Propranolol (Inderal) is appropriate because it will reduce heart rate and cardiac output.
Negative inotropism will help reduce the incidence of angina. It is an effective
antihypertensive agent
B. Propranolol (Inderal) is inappropriate because it is only useful in mild hypertension; a better
drug would be minoxidil or hydralazine because they are more effective in lowering blood
pressure
C. Propranolol (Inderal) is appropriate because it is an effective, low-cost antihypertensive. It
will augment the effects of terbutaline, an additional benefit
D. *Propranolol (Inderal) is inappropriate because its use is contraindicated in a patient with
asthma.
E. all of the above
76. Which of the following are true regarding cephalosporins? "third generation" cephalosporins are
generally more active against gramnegative organisms; cephalosporins may depress betalactamase in certain organisms; (Enterobacter, pseudomonas, serratia) causing production of the
enzyme to increase markedly; the enzyme in part B above binds to the cephalosporin;
enterococcus is never reliably sensitive to any available cephalosporin.
A. "third generation" cephalosporins are generally more active against gramnegative
organisms; cephalosporins may depress beta-lactamase in certain organisms; (Enterobacter,
pseudomonas, serratia) causing production of the enzyme to increase markedly; the enzyme
in part B above binds to the cephalosporin;
B. "third generation" cephalosporins are generally more active against gramnegative
organisms; the enzyme in part B above binds to the cephalosporin;
C. cephalosporins may depress beta-lactamase in certain organisms; (Enterobacter,
pseudomonas, serratia) causing production of the enzyme to increase markedly;
enterococcus is never reliably sensitive to any available cephalosporin.
D. enterococcus is never reliably sensitive to any available cephalosporin only
E. *All of the above
77. Aminoglycosides: are very popular; are well absorbed from the gastrointestinal tract; do not
penetrate pulmonary secretions well; do not appear in urine in high concentrations.
A. are very popular; are well absorbed from the gastrointestinal tract; do not penetrate
pulmonary secretions well
B. *are very popular; do not penetrate pulmonary secretions well
C. are well absorbed from the gastrointestinal tract; do not appear in urine in high
concentrations.
D. do not appear in urine in high concentrations only
E. All of the above
78. Some potential barriers to successful treatment of an abscess-cavitary infection with an
aminoglycoside include: inactivation of aminoglycoside by an acidic medium; binding of the
drug by pus; decreased ability of the drug to penetrate the abscess because of poor vascularity;
inactivation of aminoglycoside by alkaline medium.
A. *inactivation of aminoglycoside by an acidic medium; binding of the drug by pus; decreased
ability of the drug to penetrate the abscess because of poor vascularity
B. inactivation of aminoglycoside by an acidic medium; decreased ability of the drug to
penetrate the abscess because of poor vascularity
C. binding of the drug by pus; inactivation of aminoglycoside by alkaline medium.
D. inactivation of aminoglycoside by alkaline medium only
E. All of the above
79. Cefazolin is generally preferred among the first-generation cephalosporins, because it: has a
favorable pharmacokinetic profile; has a longer half-life than comparable drugs; may be dosed
every 8 hours; costs less for a hospital to utilize, since it may be given less frequently than
comparable drugs/
A. has a favorable pharmacokinetic profile
B. has a longer half-life than comparable drugs
C. may be dosed every 8 hours
D. costs less for a hospital to utilize, since it may be given less frequently than comparable
drugs
E. *All of the above
80. Penicillin: has a half-life of about 30 minutes; is primarily excreted renally; has an average
duration of about 26 days when administered in the; benzathine form; readily enters CSF when
meninges are normal. Choose the correct
A. *has a half-life of about 30 minutes; is primarily excreted renally; has an average duration
of about 26 days when administered in the;
B. has a half-life of about 30 minutes; is primarily excreted renally;
C. is primarily excreted renally
D. benzathine form
E. All of the above
81. Chloramphenicol is associated with: pancytopenia; gray-baby syndrome; reversible erythroidsuppression of bone marrow; renal failure?
A. *pancytopenia; gray-baby syndrome; reversible erythroid-suppression of bone marrow;
B. pancytopenia; gray-baby syndrome
C. reversible erythroid-suppression of bone marrow; renal failure
D. renal failure
E. All of the above
82. Prophylactic antibiotics in the United States are associated with: increased incidence of
infections in some cases; decreased incidence of infections in some cases; increased cost in
some cases; decreased cost in some cases.
A. increased incidence of infections in some cases
B. decreased incidence of infections in some cases
C. increased cost in some cases;
D. decreased cost in some cases
E. *All of the above
83. Vancomycin: is a treatment for clostridium difficile-induced diarrhea; is useful in treating
methicillin-resistant staphylococci; is potentially ototoxic, requires a dosage adjustment when
administered to patients with impaired renal function?
A. is a treatment for clostridium difficile-induced diarrhea; is useful in treating methicillinresistant staphylococci;
B. is potentially ototoxic
C. is useful in treating methicillin-resistant staphylococci; is potentially ototoxic
D. requires a dosage adjustment when administered to patients with impaired renal function
E. *All of the above
84. Cavalier use of antibiotics in the hospital setting may be associated with: increased cost;
increased toxicity; alteration of hospital flora; increased incidence of infections?
A. increased cost; increased toxicity;
B. increased cost; alteration of hospital flora;
C. alteration of hospital flora; increased incidence of infections
D. increased incidence of infections
E. *All of the above
85. Imipenem is a beta-lactam antibiotic which is neither a penicillin nor a cephalosporin. Correct
statements regarding imipenem include: it covers an extremely broad spectrum of
microorganisms; it is very active against many gramnegative rods; resistant pseudomonas may
emerge during treatment; it should not be given to patients having a history of allergic reactions
to penicillin
A. it covers an extremely broad spectrum of microorganisms; it is very active against many
gramnegative rods
B. resistant pseudomonas may emerge during treatment
C. it is very active against many gramnegative rods
D. it should not be given to patients having a history of allergic reactions to penicillin
E. *All of the above
86. Patients having a history of a severe, immediate reaction to penicillin: may be given a
cephalosporin without concern; have a definite risk of reaction to any cephalosporin; have a
low risk of having a reaction to a broad spectrum antipseudomonal penicillin; have a high risk
of hypersensitivity to a broad spectrum anti-pseudomonal penicillin
A. may be given a cephalosporin without concern; have a definite risk of reaction to any
cephalosporin;
B. may be given a cephalosporin without concern; have a low risk of having a reaction to a
broad spectrum antipseudomonal penicillin
C. *have a definite risk of reaction to any cephalosporin; have a high risk of hypersensitivity to
a broad spectrum anti-pseudomonal penicillin
D. have a high risk of hypersensitivity to a broad spectrum anti-pseudomonal penicillin
E. All of the above
87. A ninety two year-old female was brought to the emergency room following an episode of low
blood pressure and disorientation at the nursing home. She had been refusing food and drank
little. She was provided i.v glucose and was rehydrated. Over the next few days her condition
worsened with increasing periods of incoherence. She exhibited hyperventilation with
respiratory alkalosis and pulmonary congestion. Her body temperature was slightly subnormal
and she was hypotensive (75/40 mm Hg) and tachycardic. Her pulmonary congestion worsened
as did her blood gases. Digoxin was administered. Blood cultures indicated the present of E.
coli, leading to a definitive diagnosis of E. coli septicemia. Which drug(s) would be appropriate
to treat the septicemia?
A. cefotaxime (Claforan)
B. ceftriaxone (Rocephin)
C. aztreonam (Azactan)
D. *all of the above
E. None of the above
88. A sixty-year old male was recovering from bowel resection. The initial recovery period was
unremarkable. Late on the third post-operative day the patient began hyperventilating with
respiratory alkalosis. On the fourth post-operative day, the patient's condition rapidly worsened
with a significant elevation of temperature (104.9 F), profound hypotension (65/38 mm Hg) and
tachycardia. Urine output was very low with BUN and serum creatinine (5 mg%, which had
increased from normal levels at admission) were elevated. Gram staining of sputum, which was
purulent, indicated gram-negative rods. WBC was 17000/mm, septic shock with a shift to the
left. The diagnosis was fulminant bacteremia and septic shock. What mechanism is most likely
responsible for this patient's tachycardia?
A. Direct effects of bacterial endotoxins on the SA node
B. *Significant hypotension associated with low systemic vascular resistance.
C. bacterial endotoxins bind to muscarinic cholinergic receptors, thus removing the dominating
cholinergic tone in the heart.
D. patient anxiety
E. In direct effect of bacterial endotoxin on the SA node
89. A 23 years old patient is pregnant with a history of severe (anaphylactic) penicillin allergy. To
prepare her for an upcoming tooth extraction you prescribe an antimicrobial medication that can
be taken prophlaxis 2 hrs prior to the procedure. This medication is:
A. Cefaclor
B. Doxycycline
C. Erythromycin base
D. *Erythromycin stearate
E. Gentamycin
90. Mr.Antee Biatek, a 30 yr old woman is scheduled to undergo an exploration laparotomy to
confirm the diagnosis of an ectopic pregnancy. She has a history of several penicillin allergy.
For prophylaxis prior to her surgery you administer:
A. *Chloramphenicol
B. Erythromycin stearate
C. Gentamycin + vancomycin i.v
D. Rifampin
E. Streptogramins A+B (synercid)
91. Amphotericin B is a very useful agent for treating fungal infections. Correct statements
regarding amphotericin include: it is both hydrophilic and lipophilic; it binds to sterols in the
fungal cell membrane; a test dose should be administered before treatment begins;
nephrotoxicity may occur?
A. it is both hydrophilic and lipophilic; it binds to sterols in the fungal cell membrane;
B. it binds to sterols in the fungal cell membrane; nephrotoxicity may occur
C. it is both hydrophilic and lipophilic; a test dose should be administered before treatment
begins;
D. nephrotoxicity may occur
E. *All of the above
92. Sulfonamides: are competitive antagonists of PABA, and thereby decrease bacterial utilization
of para-aminobenzoic acid in the synthesis of folic acid; do not affect mammalian cells, since
they require pre-formed folic acid; are synergistic with trimethoprim, which inhibits
dihydrofolate reductase (which reduces dihydrofolate to tetrahydrofolate); can be antagonized
by PABA?
A. are competitive antagonists of PABA, and thereby decrease bacterial utilization of paraaminobenzoic acid in the synthesis of folic acid; do not affect mammalian cells, since they
require pre-formed folic acid; are synergistic with trimethoprim, which inhibits
dihydrofolate reductase (which reduces dihydrofolate to tetrahydrofolate)
B. are competitive antagonists of PABA, and thereby decrease bacterial utilization of paraaminobenzoic acid in the synthesis of folic acid; are synergistic with trimethoprim, which
inhibits dihydrofolate reductase (which reduces dihydrofolate to tetrahydrofolate)
C. do not affect mammalian cells, since they require pre-formed folic acid; can be antagonized
by PABA
D. can be antagonized by PABA only
E. *All of the above
93. Circumstances associated with allergy to penicillins may include: maculopapular or urticarial
rash; anaphylaxis; anaphylactic reaction to penicillin skin-tests; exposure to penicillins in
food?
A. maculopapular or urticarial rash
B. maculopapular or urticarial rash; anaphylactic reaction to penicillin skin-tests;
C. Anaphylaxis
D. exposure to penicillins in food
E. *All of the above
94. Aminoglycosides: must be dosed on the basis of renal function; are primarily active against
gram-negative aerobes; are not active against anaerobes; penicillin may be synergistic against
enterococcus?
A. must be dosed on the basis of renal function; are primarily active against gram-negative
aerobes; are not active against anaerobes
B. must be dosed on the basis of renal function; are not active against anaerobes
C. are primarily active against gram-negative aerobes; penicillin may be synergistic against
enterococcus
D. penicillin may be synergistic against enterococcus only
E. *All of the above
95. A week after initiating clindamycin therapy the patient develops signs of a potentially fatal
infection of the colon. His tests reveal that his infection is due to clostridium difficile, to treat
this new condition you should initiate drug therapy with
A. a higher dose of clindamycin
B. gentamicin i.v
C. linezolid
D. *metronidazole
E. rifampin
96. Imipenem is a beta-lactam antibiotic which is neither a penicillin nor a cephalosporin. Correct
statements regarding imipenem include: it covers an extremely broad spectrum of
microorganisms; it is very active against many gram negative rods; resistant pseudomonas may
emerge during treatment; it should not be given to patients having a history of allergic reactions
to penicillin?
A. it covers an extremely broad spectrum of microorganisms; it is very active against many
gram negative rods; resistant pseudomonas may emerge during treatment;
B. it covers an extremely broad spectrum of microorganisms; resistant pseudomonas may
emerge during treatment;
C. it is very active against many gram negative rods; it should not be given to patients having a
history of allergic reactions to penicillin
D. it should not be given to patients having a history of allergic reactions to penicillin only
E. *All of the above
97. Which of the following are true regarding cephalosporins? "third generation" cephalosporins
are generally more active against gram-negative organisms; cephalosporins may depress betalactamase in certain organisms (Enterobacter, pseudomonas, serratia) causing production of the
enzyme to increase markedly; the enzyme in part B above binds to the cephalosporin;
enterococcus is never reliably sensitive to any available cephalosporin
A. "third generation" cephalosporins are generally more active against gram-negative
organisms
B. cephalosporins may depress beta-lactamase in certain organisms (Enterobacter,
pseudomonas, serratia) causing production of the enzyme to increase markedly
C. the enzyme in part B above binds to the cephalosporin;
D. enterococcus is never reliably sensitive to any available cephalosporin
E. *All of the above
98. Prophylactic antibiotics in the United States are associated with: increased incidence of
infections in some cases; decreased incidence of infections in some cases; increased cost in
some cases; decreased cost in some cases?
A. increased incidence of infections in some cases; decreased incidence of infections in some
cases; increased cost in some cases
B. increased incidence of infections in some cases; increased cost in some cases
C. decreased incidence of infections in some cases; decreased cost in some cases
D. decreased cost in some cases only
E. *All of the above
99. Penicillin: has a half-life of about 30 minutes; is primarily excreted renally; has an average
duration of about 26 days when administered in the benzathine form; readily enters CSF when
meninges are normal?
A. *has a half-life of about 30 minutes; is primarily excreted renally; has an average duration of
about 26 days when administered in the benzathine form
B. has a half-life of about 30 minutes; has an average duration of about 26 days when
administered in the benzathine form
C. is primarily excreted renally; readily enters CSF when meninges are normal
D. readily enters CSF when meninges are normal
E. All of the above
100. Vancomycin: is a treatment for clostridium difficile-induced diarrhea; is useful in treating
methicillin-resistant staphylococci; is potentially ototoxic; requires a dosage adjustment when
administered to patients with impaired renal function?
A. is a treatment for clostridium difficile-induced diarrhea; is useful in treating methicillinresistant staphylococci; is potentially ototoxic
B. is a treatment for clostridium difficile-induced diarrhea; is potentially ototoxic
C. is useful in treating methicillin-resistant staphylococci; requires a dosage adjustment when
administered to patients with impaired renal function
D. requires a dosage adjustment when administered to patients with impaired renal function
only
E. *All of the above
101. Cefazolin is generally preferred among the first-generation cephalosporins, because it: has a
favorable pharmacokinetic profile; has a longer half-life than comparable drugs; may be dosed
every 8 hours; costs less for a hospital to utilize, since it may be given less frequently than
comparable drugs?
A. has a favorable pharmacokinetic profile;
B. has a longer half-life than comparable drugs; may be dosed every 8 hours
C. has a longer half-life than comparable drugs;costs less for a hospital to utilize, since it may
be given less frequently than comparable drugs
D. costs less for a hospital to utilize, since it may be given less frequently than comparable
drugs
E. *All of the above
102. Patients having a history of a severe, immediate reaction to penicillin: may be given a
cephalosporin without concern; have a definite risk of reaction to any cephalosporin; have a low
risk of having a reaction to a broad spectrum antipseudomonal penicillin; have a high risk of
hypersensitivity to a broad spectrum anti-pseudomonal penicillin
A. may be given a cephalosporin without concern; have a definite risk of reaction to any
cephalosporin; have a low risk of having a reaction to a broad spectrum antipseudomonal
penicillin
B. may be given a cephalosporin without concern; have a low risk of having a reaction to a
broad spectrum antipseudomonal penicillin
C. *have a definite risk of reaction to any cephalosporin; have a high risk of hypersensitivity to
a broad spectrum anti-pseudomonal penicillin
D. have a high risk of hypersensitivity to a broad spectrum anti-pseudomonal penicillin only
E. All of the above
103. Amphotericin B is a very useful agent for treating fungal infections. Correctstatements
regarding amphotericin include: it is both hydrophilic and lipophilic; it binds to sterols in the
fungal cell membrane; a test dose should be administered before treatment begins;
nephrotoxicity may occur
A. it is both hydrophilic and lipophilic; it binds to sterols in the fungal cell membrane; a test
dose should be administered before treatment begins
B. it is both hydrophilic and lipophilic; a test dose should be administered before treatment
begins
C. it binds to sterols in the fungal cell membrane; nephrotoxicity may occur
D. nephrotoxicity may occur only
E. *All of the above
104. Some potential barriers to successful treatment of an abscess-cavitary infection with an
aminoglycoside include: inactivation of aminoglycoside by an acidic medium; binding of the
drug by pus; decreased ability of the drug to penetrate the abscess because of poor vascularity;
inactivation of aminoglycoside by alkaline medium?
A. *inactivation of aminoglycoside by an acidic medium; binding of the drug by pus;
decreased ability of the drug to penetrate the abscess because of poor vascularity;
B. inactivation of aminoglycoside by an acidic medium
C. binding of the drug by pus; inactivation of aminoglycoside by alkaline medium
D. inactivation of aminoglycoside by alkaline medium
E. All of the above
105. True statement concerning the pharmacologic effects of salicylates include: salicylates are
thought to exert their activity at least partially by inhibiting prostaglandin synthetase; high-dose
salicylate therapy (more than 5 g /day) lowers the serum uric acid concentration; the effect of
salicylates upon platelet aggregation is irreversible unlike that of other non-steroidal antiinflammatory drugs; salicylate overdose is potentionally fatal, however, promt administration
of acetylcystein will avert this danger?
A. *salicylates are thought to exert their activity at least partially by inhibiting prostaglandin
synthetase; high-dose salicylate therapy (more than 5 g /day) lowers the serum uric acid
concentration; the effect of salicylates upon platelet aggregation is irreversible unlike that
of other non-steroidal anti-inflammatory drugs
B. salicylates are thought to exert their activity at least partially by inhibiting prostaglandin
synthetase; high-dose salicylate therapy (more than 5 g /day) lowers the serum uric acid
concentration;
C. the effect of salicylates upon platelet aggregation is irreversible unlike that of other nonsteroidal anti-inflammatory drugs
D. salicylate overdose is potentionally fatal, however, promt administration of acetylcystein
will avert this danger
E. All above
106. A doctor is caring for a client with a diagnosis of rheumatoid arthritis who is receiving
sulindac (Clinoril) 150 mg po twice daily. Which finding would indicate to the doctor that the
client is experiencing a side effect related to the medication?
A. *nausea
B. photophobia
C. fever
D. tingling in the extremities
E. All of the above
107. Angina pectoris is:
A. *Severe constricting chest pain, often radiating from the precordium to the left shoulder and
down the arm, due to insufficient blood supply to the heart that is usually caused by
coronary disease
B. An often fatal form of arrhythmia characterized by rapid, irregular fibrillar twitching of the
ventricles of the heart instead of normal contractions, resulting in a loss of pulse
C. The cardiovascular condition in which the heart ability to pump blood weakens
D. All of the above
E. None of the above
108. All these drug groups useful in angina both decrease myocardial oxygen requirement (by
decreasing the determinations of oxygen demand) and increase myocardial oxygen delivery (by
reversing coronary arterial spasm), EXCEPT:
A. Nitrates and nitrite drugs (Nitroglycerin, Isosorbide dinitrate)
B. Calcium channel blockers (Nifedipine, Nimodipine)
C. *Beta-adrenoceptor-blocking drugs (Atenolol, Metoprolol)
D. Potassium channel openers (Minoxidil)
E. None of the above
109. This drug group useful in angina decreases myocardial oxygen requirement (by decreasing
the determinations of oxygen demand) and does not increase myocardial oxygen delivery (by
reversing coronary arterial spasm):
A. Nitrates and nitrite drugs (Nitroglycerin, Isosorbide dinitrate)
B. Myotropic coronary dilators (Dipyridamole)
C. Potassium channel openers (Minoxidil)
D. *Beta-adrenoceptor-blocking drugs (Atenolol, Mtoprolol)
E. None of the above
110. This drug group useful in angina increase myocardial oxygen delivery (by reversing
coronary arterial spasm) and does not decrease myocardial oxygen requirement (by decreasing
the determinations of oxygen demand):
A. Beta-adrenoceptor-blocking drugs (Atenolol, Metoprolol):
B. *Myotropic coronary dilators (Dipyridamole)
C. Calcium channel blockers (Nifedipine, Nimodipine)
D. Potassium channel openers (Minoxidil)
E. All of the above
111. Which of the following statements concerning nitrate mechanism of action is TRUE?
A. Therapeutically active agents in this group are capable of releasing nitric oxide (NO) in to
vascular smooth muscle target tissues
B. Nitric oxide (NO) is an effective activator of soluble guanylyl cyclase and probably acts
mainly through this mechanism
C. Nitrates useful in angina decrease myocardial oxygen requirement (by decreasing the
determinations of oxygen demand) and increase myocardial oxygen delivery (by reversing
coronary arterial spasm)
D. *All of the above
E. None of the above
112. Which of the following nitrates and nitrite drugs are long-acting?
A. Nitroglycerin, sublingual
B. Isosorbide dinitrate, sublingual (Isordil, Sorbitrate)
C. Amyl nitrite, inhalant (Aspirols, Vaporole)
D. *Sustac
E. All of the above
113. Which of the following nitrates and nitrite drugs is a short-acting drug?
A. Nitroglycerin, 2% ointment (Nitrol)
B. Nitroglycerin, oral sustained-release (Nitrong)
C. *Amyl nitrite, inhalant (Aspirols, Vaporole)
D. Sustac
E. None of the above
114. Which of the following nitrates and nitrite drugs is used for prevention of angina attack?
A. Nitroglycerin, 2% ointment (Nitrol)
B. Nitroglycerin, oral sustained-release (Nitrong)
C. Isosorbide mononitrate (Ismo)
D. *All of the above
E. None of the above
115. Duration of nitroglycerin action (sublingual) is:
A. *10-30 minutes
B. 6-8 hours
C. 3-5 minutes
D. 1.5-2 hours
E. 2-3 hours
116. The following statements concerning mechanism of nitrate beneficial clinical effect are true,
EXCEPT?
A. Decreased myocardial oxygen requirement
B. Relief of coronary artery spasm
C. Improved perfusion to ischemic myocardium
D. Diminishes preload (venous return to the heart)
E. *Increased myocardial oxygen consumption
117. Side effect of nitrates and nitrite drugs are, EXCEPT:
A. Orthostatic hypotension,
B. Tachycardia
C. *GI disturbance
D. Throbbing headache
E. Facial flushing
118. The following statements concerning mechanism of calcium channel blockers’ action are
true, EXCEPT:
A. *Therapeutically active agents in this group are capable of releasing nitric oxide (NO) in
vascular smooth muscle target tissues
B. Calcium channel blockers bind to L-type calcium channel sites
C. Calcium channel blockers useful in angina decrease myocardial oxygen requirement (by
decreasing the determinations of oxygen demand)
D. Calcium channel blockers increase myocardial oxygen delivery (by reversing coronary
arterial spasm)
E. Calcium channel blockers decrease transmembrane calcium current associated in smooth
muscle with long-lasting relaxation and in a cardiac muscle with a reduction in contractility
119. Which of the following antianginal agents is a calcium channel blocker?
A. Nitroglycerin
B. Lidocaine
C. Dipyridamole
D. Minoxidil
E. *Nifedipine
120. Which of the following cardiovascular system effects refers to a calcium channel blocker?
A. The reduction of peripheral vascular resistance
B. The reduction of cardiac contractility and, in some cases, cardiac output
C. Relief of coronary artery spasm
D. *All of the above
E. None of the above
121. Main clinical use of calcium channel blockers is:
A. Angina pectoris
B. Hypertension
C. Supraventricular tachyarrhythmias
D. None of the above
E. *All of the above
122. Which of the following antianginal agents is a myotropic coronary dilator:
A. *Dipyridamole
B. Validol
C. Atenolol
D. Alinidine
E. Sotalol
123. Which of the following antianginal agents is a beta-adrenoceptor-blocking drug:
A. Dipyridamole
B. Validol
C. *Metoprolol
D. Alinidine
E. Salmeterol
124. The following agents are cardioselective beta1-adrenoceptor-blocking drugs labeled for use
in angina, EXCEPT:
A. Metoprolol
B. Talinolol
C. Atenolol
D. *Propranolol
E. Carvedilol
125. Which of the following antianginal agents refers to reflex coronary dilators:
A. Dipyridamole
B. *Validol
C. Atenolol
D. Alinidine
E. Molsydomin
126. Which of the following statements concerning Validol is true:
A. Validol has a moderate reflex and vascular dilative action caused by the stimulation of
sensitive nerve endings
B. At sublingual administration the effect is produced in five minutes and 70 % of the
preparation is released in 3 minutes
C. It is used in cases of angina pectoris, motion sickness, nausea, vomiting when seasick or
airsick and headaches due to taking nitrates
D. *All of the above
E. None of the above
127. Which of the following antianginal agents is a potassium channel opener:
A. Dipyridamole
B. Validol
C. Atenolol
D. *Minoxidil
E. Lisinopril
128. A man aged 74 has moderate hypertension controlled with hydrochlorothiazide 12.5 mg
once daily and losartan 50 mg once daily. He is prescribed rofecoxib 50 mg once daily to
control osteoarthritis pain. After 3 months of this therapy, his blood pressure begins to rise.This
increase in blood pressure is most likely due to
A. *Inhibition of COX-2 by rofecoxib, which leads to decreased renal blood flow
B. Increased metabolism of losartan due to induction of CYP2C9 by rofecoxib
C. Increased excretion of hydrochlorothiazide due to increased renal blood flow caused by
rofecoxib
D. Arteriolar contraction in the peripheral circulation caused by inhibition of COX-1 by
rofecoxib
E. Weight gain caused by rofecoxib’s ability to decrease basal metabolic rate.
129. A 52-year-old woman with a history of eczema and heavy alcohol use begins taking
ibuprofen to control hip and knee pain due to osteoarthritis. Over the course of 6 months, as the
pain worsens, she increases her dosage to a high level (600 mg four times daily).What toxicity is
most likely to occur, and why?
A. Abnormal heart rhythms; alcohol induces cytochrome P450 isozymes that convert ibuprofen
to a cardiotoxic free radical metabolite
B. Necrotizing fasciitis; eczema predisposes an individual to this toxicity of ibuprofen
C. *Gastric ulceration; heavy alcohol use increases the susceptibility of an individual to
ibuprofeninduced GI toxicity
D. Confusion and ataxia; these CNS toxicities of ibuprofen are additive with those of ethanol
E. Eosinophilia; this rare complication of ibuprofen therapy is exacerbated by the
immunosuppression frequently seen in alcoholics
130. A patient refuses to continue to take erythromycin because it makes him vomit.This is an
example of which patient–drug–pathogen interaction?
A. Pharmacokinetics
B. *Pharmacodynamics
C. Immunity
D. Resistance
E. Selective toxicity
131. A 24-year-old AIDS patient is interested in starting chemoprophylaxis for Pneumocystis
pneumonia (PCP) and cerebral toxoplasmosis. He has no drug allergies.Which of the following
prophylactic agents is appropriate for the prevention of both PCP and cerebral toxoplasmosis?
A. Nitrofurantoin
B. *Trimethoprim–sulfamethoxazole
C. Norfloxacin
D. Methenamine
E. Nalidixic acid
132. Urinalysis of a 38-year-old woman with recurrent UTIs revealed pH 6.8, 30 to 50 WBC per
highpower field, and gram-negative bacilli identified as Proteus mirabilis.Which of the
following produces a bacteriostatic urinary environment for P. mirabilis?
A. Urease enzyme
B. *Hippuric acid
C. Catalase enzyme
D. Folic acid
E. Coagulase enzyme
133. A 3-day-old baby is given a presumptive diagnosis of kernicterus.Which of the following
mechanisms is involved in sulfonamide-induced kernicterus?
A. *Competes for the bilirubin-binding sites on plasma proteins
B. Defective bilirubin hepatic conjugation and metabolism
C. Physiological jaundice due to destruction of fetal red blood mass
D. Pregnancy-induced hepatic congestion and cholestasis
E. Primary biliary cirrhosis of the liver
134. A 6-year-old relatively healthy boy is diagnosed with external otitis and was prescribed a 7day course of TMP-SMX.Which of the following is the basic mechanism of action of the
sulfonamides?
A. Selective inhibition of incorporation of PABA into human cell folic acid synthesis.
B. *Competitive inhibition of incorporation of PABA into microbial folic acid.
C. Inhibition of transpeptidation reaction in bacterial cell wall synthesis.
D. Changes in DNA gyrases and active efflux transport system resulting in decreased
permeability of drug.
E. Structural changes in dihydropteroate synthase and overproduction of PABA.
135. Evaluation of a yearly chest radiograph of a 73-year-old patient taking nitrofurantoin
prophylactically for recurrent UTIs revealed new findings of bilateral interstitial fibrosis. What
is the possible explanation for the patient’s pulmonary presentation and what is the next step?
A. Acute urosepsis; add a broad-spectrum antibiotic to nitrofurantoin.
B. *Possible allergic reaction to nitrofurantoin; stop it immediately.
C. Nitrofurantoin-resistant E. coli infection; stop it immediately.
D. Acute community-acquired streptococcal pneumonia; treat accordingly.
E. Nitrofurantoin-induced hemolysis; requires permanent urinary catheter.
136. A 16-year-old girl, a cystic fibrosis patient, is diagnosed with a ciprofloxacin-resistant
Pseudomonas aeruginosa lower respiratory tract infection. Bacteria acquire quinolone
resistance by which of the following mechanisms?
A. Overproduction of PABA
B. *Changes in the synthesis of DNA gyrases
C. Plasmid-mediated changes in efflux transport system
D. Inhibition of synthesis of peptidoglycan subunits in bacterial cell walls
E. Inhibition of folic acid synthesis by blocking different steps
137. Lipoprotein is:
A. *A conjugated protein having a lipid component; the principal means for transporting lipids
in the blood
B. Any of various fat-soluble or water-soluble organic substances essential in minute amounts
for normal growth and activity of the body and obtained naturally from plant and animal
foods
C. Product of endocrine gland secretion
D. Mediators of inflammatory process
E. None of the above
138. Very low density lipoprotein (VLDL) is:
A. *A lipoprotein containing a very large proportion of lipids to protein and carrying most
cholesterol from the liver to the tissues
B. A lipoprotein that contains relatively high amounts of cholesterol and is associated with an
increased risk of atherosclerosis and coronary artery disease. It is also called betalipoprotein
C. A lipoprotein that contains relatively small amounts of cholesterol and triglycerides and is
associated with a decreased risk of atherosclerosis and coronary artery disease. It is also
called alpha-lipoprotein
D. Large lipoprotein particle that is created by the absorptive cells of the small intestine. It
transports lipids to adipose tissue where they are broken down by lipoprotein lipase
E. None of the above
139. Low-density lipoprotein (LDL) is:
A. *A lipoprotein that contains relatively high amounts of cholesterol and is associated with an
increased risk of atherosclerosis and coronary artery disease. It is also called betalipoprotein
B. A lipoprotein that contains relatively small amounts of cholesterol and triglycerides and is
associated with a decreased risk of atherosclerosis and coronary artery disease. It is also
called alpha-lipoprotein
C. A lipoprotein containing a very large proportion of lipids to protein and carrying most
cholesterol from the liver to the tissues
D. Large lipoprotein particle that is created by the absorptive cells of the small intestine. It
transports lipids to adipose tissue where they are broken down by lipoprotein lipase
E. None of the above
140. High-density lipoprotein (HDL) is:
A. *A lipoprotein that contains relatively small amounts of cholesterol and triglycerides and is
associated with a decreased risk of atherosclerosis and coronary artery disease. It is also
called alpha-lipoprotein
B. A lipoprotein containing a very large proportion of lipids to protein and carrying most
cholesterol from the liver to the tissues
C. A lipoprotein that contains relatively high amounts of cholesterol and is associated with an
increased risk of atherosclerosis and coronary artery disease. It is also called betalipoprotein
D. Large lipoprotein particle that is created by the absorptive cells of the small intestine. It
transports lipids to adipose tissue where they are broken down by lipoprotein lipase
E. None of the above
141. Hyperlipoproteinemia
A. *is a condition marked by an abnormally high level of lipoproteins in the blood.
B. Is high blood levels of triglycerides. It has been associated with atherosclerosis
C. is an abnormally high concentration of cholesterol in the blood.
D. is an abnormally high concentration of low-density lipoprotein in the blood
E. None of the above
142. All of the following statements concerning drugs which inhibit cholesterol synthesis are
true, EXCEPT:
A. They work in part by increasing the rate of LDL clearance from the plasma
B. They are the most effective single agents for lowering LDL-cholesterol
C. When used with a bile-acid binding resin, they can lower LDL-cholesterol by 50% or more
D. *No special monitoring is required in patients receiving one of them
E. None of the above
143. All of the following statements concerning nicotinic acid (Niacin) are true, EXCEPT:
A. It reduces the rate of synthesis of VLDL(very low-density lipoprotein)
B. *Sustained-release preparations of this drug are largely free of side effects
C. Almost all patients taking the traditional dosage form of this drug experience uncomfortable
flushing
D. It should not be used with antihypertensives
E. All of the above are true
144. All of the following statements concerning drugs which inhibit cholesterol synthesis are
true, EXCEPT:
A. These drugs should not be used in pregnant women or children
B. These drugs often cause myopathy if used in combination with cyclosporine (Sandimmune)
C. Failure to discontinue the drug after myopathy has been detected can cause acute renal
failure
D. *Several of these drugs tend to lengthen the sleep cycle
E. All of the above are true
145. All of the following statements concerning the general principles of therapy with lipidlowering drugs are true EXCEPT:
A. Therapy with a lipid-lowering drug should be always accompanied by an appropriate diet
B. *A lipid-lowering diet should be discontinued if it fails to decrease the levels of plasma LDL
cholesterol by at least 10%
C. Lipid-lowering drugs should only be administered after at least 3 months of prior dietary
therapy
D. Some combinations of lipid-lowering drugs are synergistic
E. All of the above are true
146. This drug can cause muscle damage, especially when used with any of several drugs
including erythromycin:
A. Cholestyramine (Questran)
B. *Lovastatin (Mevacor)
C. Gemfibrozil (Loprol)
D. Probucol (Lorelco)
E. Clofibratum
147. This fibric acid derivative increases blood levels of high density lipoproteins (HDL):
A. Cholestyramine (Questran)
B. Lovastatin (Mevacor)
C. Atorvastatin
D. *Gemfibrozil (Loprol)
E. Probucol (Lorelco)
148. Which of the following drugs used in the treatment of gout has as its primary effect the
reduction of uric acid synthesis
A. *Allopurinol
B. Sulfinpyrazone
C. Colchicine
D. Indomethacin
E. Cyclosporine
149. Which of the following statements refers to 1,25-dihydroxyvitamin D3 (calcitriol):
A. *The combined effect of calcitriol and all other vitamin D metabolites and analogs on both
calcium and phosphate makes careful monitoring of the level of these minerals especially
important to avoid ectopic calcification
B. Does not undergo enterohepatic circulation
C. Toxic to osteoclasts
D. Bioavailability increases with the administered dose
E. All of the above
150. Correct statements about fluoride include all of the following, EXCEPT:
A. Fluoride is effective for the prophylaxis of dental caries
B. Fluoride is accumulated by bone and teeth, where it may stabilize the hydroxyapatite crystal
C. *Subjects living in areas with naturally fluoridated water (1-2 ppm) had more dental caries
and fewer vertebral compression fractures than subjects living in nonfluoridated water areas
D. Chronic exposure to very high level of fluoride dust in the inspired air results in crippling
fluorosis, characterized by thickening of the cortex of long bones and bony exostoses.
E. All of the above